TestPrep (P) - Medical

अब Quizwiz के साथ अपने होमवर्क और परीक्षाओं को एस करें!

When scoring deep te ndon reflexes, a grade of 2+ indicates which of the following? A) Active B) Sluggish C) Hyperactive D) Slightly hyperactive

A) Active Feedback: As part of an inclusive neurologic assessment, and when time and patient condition permit, the paramedic should assess and document the patient's deep tendon reflexes (DTRs). Grade O indicates no response, Grade I+ indicates sluggish reflexes, Grade 2+ indicates active (normal) reflexes, Grade 3+ indicates slightly hyperactive reflexes, and Grade 4+ indicates hyperactive reflexes. Hyperreflexia can be caused by conditions such as preeclampsia, multiple sclerosis, hypothyroidism, and Reye syndrome, among others. Hyporeflexia can be caused by conditions such as amyotrophic lateral sclerosis (Lou Gehrig disease), hypothyroidism, and spinal cord injuries, among od1ers.

An older woman presents with severe weakness, hypotension, lower back pain, and vomiting. Her husband tells you that she has not taken her prednisone in several days because she has not been feeling well. What should you suspect? A) Addisonian crisis B) Cushing syndrome C) Pheochromocytoma D) Thyrotoxic crisis

A) Addisonian crisis Feedback: Signs and symptoms of acute adrenal insufficiency can manifest suddenly in what is called an addisonian crisis. Abrupt cessation of corticosteroid therapy (ie, prednisone, hydrocortisone) is the most common cause of an addisonian crisis. It may also be triggered by acute exacerbation of chronic adrenal insufficiency (Addison disease), usually brought on by stress, trauma, surgery, or a severe in fection. In either case, cardiovascular collap e occurs due to a lack of the hormone cortisol; therefore, the chief clinical manifestation of an addisonian crisis is shock. Other signs and symptoms may include weakness; lethargy; fever; severe pain in the lower back, legs, or abdomen; and severe vomiting and diarrhea. C ushing syndrome is caused by excessive cortisol production by the adrenal cortex; it may also occur if large amounts of corticosteroids are administered. Pheochromocytoma is an adrenal tumor that causes excessive release of ep inephrine and norepinephrine; patients with this condition present with hypertension and tachycardia. Thyrotoxic crisis (thyroid storm) is a condition caused by critically high thyroid horn1one levels, resulting in a hypermetabolic state. Signs and symptoms include evere tachycardia, hypertension, fever, altered mental status, and possibly heart failure.

Which of the following can be assessed without talking to a patient? A) Affect B) Memory C) Thought D) Orientation

A) Affect Feedback: Mood and affect can be assessed through visual obse,vation. Mood can be objectively noted via the patient's body language to determine if it is euphoric, sad, or labile. Affect-the expression of i n ner feelings-can be assessed by noting if it seems appropriate to the situation, is ani mated, angry, flat, or withdrawn. Assessment of memory, thought, and orientation require you to ask the patient direct questions. Si mply observing the patient and listening to them can yield a lot of information, such as the potential for violence (ie, increased tone of voice, pressured speech, clenched fists), w hen assessing a patient with a behavioral crisis.

Impairment of proprioception increases the risk of which of the following in older adults? A) Falls B) Stroke C) Blindness D) Heart attack

A) Falls Feedback: Proprioception is the ability to perceive the position and movement of one's body or limbs. It enables a person to maintain postural stability using a variety of receptors in the joints and information provided by the eyes. As these mechanisms decrease with age, people become less steady on their feet, and the tendency to fall increases markedly. Other changes in sensory perception also increase the risk for falls. Conditions such as atherosclerosis and arteriosclerosis increase a person's risk for heart attack and stroke. Conditions such as macular degeneration and glaucoma can have an impact on vision.

Which of the following hormones stim ulates the kidneys to reabsorb sodium and excrete potassium? A) Aldosterone B) Somatostatin C) Antidiuretic hormone D) Adrenocorticotropic hormone

A) Aldosterone Feedback: If the body experiences a drop in blood pressure or volume, a decrease in sodium levels, or an increase in potassium levels, the adrenal cortex i stimulated to secrete aldosterone (a mineralocorticoid). Aldosterone stimulates the kidneys to reabsorb sodi um from the urine and excrete potassium by a ltering the osmotic gradient in the blood. When sodium is reabsorbed into the blood, water follows; this action increases both blood volume and pressure. Somatostatin is a hormone secreted by the delta cells of the pancreas; it inhibits insulin and glucagon secretion. Antidiuretic hormone (ADH), which is released by the pituitary gland, regulates water balance in the body. I f a person is dehydrated, ADH secretion increases, which stimulates the renal tubules to reabsorb sodium and water and inhibits diu resis. If a person is overhydrated, ADH secretion decreases, thereby faci litating diuresis. Adrenoconicotropic hormone (ACTH) is secreted by the pituitary gland; it stimulates the adrenal cortex to manufacture and secrete cortisol.

A man reports burning in his throat and a severe headache after being rescued from his burning house. His BP is 180/90 mm Hg, pulse is 120 beats/min, and respirations are 28 breaths/min and labored. What treatment is indicated for him? A) Amyl nitrite inhaled in 20-second increments B) Rapid transport to a local hyperbaric facility C) 1 to 2 g of calcium chloride slow IV push D) 1 mL/kg of ethyl alcohol slow IV push

A) Amyl nitrite inhaled in 20-second increments Feedback: A burni ng sensation in the mouth or throat, headache, and sympathetic hyperactivity (hypertension and tachycardia) are indicative of cyanide toxicity. The odor of bitter almonds on the patient's breath, while highly suggestive of cyanide toxicity, is not diagnostic. Cyanide acts as a cellular asphyxiant, inhibiting oxygen uptake by the tissues. This causes severe hypoxia, even in the presence of oxygen. After removing the patient from the source, and after ensuring airway patency and breathing adeq uacy, treatment includes having the patient inhale amyl nitrite for 20 seconds, followed by high-now oxygen for 40 seconds. Thus, in each minute, the patient should inhale the amyl nitrite one-third of the time and breathe o»-ygen two-thirds of the t ime. Keep switching the amyl nitrite and oxygen while maintaining this ratio. Many of the same burning chemicals that produce carbon monoxide (CO) produce cyanide. Treatment for CO exposure includes high-now oxygen and possibly hyperbaric therapy, depending on the patient's carboxyhemoglobin (COHb) level. Calcium chloride is given for hyperkalemia and calcium channel blocker overdose; it is not indicated for cyanide toxicity. Ethyl alcohol (ethanol) may be given at the hospital to treat ethylene glycol (ie, antifreeze) ingestion; it is not given for cyanide toxici ty.

Treatment for cutaneous exposure to hydrofluoric acid includes which of the following? A) Antacid preparations, such as Mylanta or Maalox B) 20 mL of a 10% solution of calcium gluconate IV C) Baking powder covered with a dry, sterile dressing D) 1 to 2 g of magnesium sulfate via slow IV push

A) Antacid preparations, such as Mylanta or Maalox Feedback: Hydrofluoric acid is one of the strongest inorganic acids. It is mainly used for industrial pu rposes, such as glass etching and metal cleaning; it may also be found in home rust removers. Tissue damage following exposure to hydrofluoric acid occurs via two mechanisms: corrosive burns from the free hydrogen ions and chemical burns from tissue penetration of the fluoride ions. Fluoride ions penetrate the skin and form insoluble salts with calcium and magnesium. Soluble salts are also formed with other cations; however, they rapidly dissociate. As a result, fluoride ions release and further tissue damage occurs. Significant exposure can re ult in hypocalcemia, hypomagnesemia, and hyperkalemia. After ensuring your own safety and properly decontaminating the patient by irrigating the affected area with water, cutaneous (skin) exposure to hydrofluoric acid is neutralized with one of two agents: calcium gl uconate gel (usually not available in the prehospital setting) or a magnesium hydroxide antacid preparation (ie, milk of magnesia, Mylanta, Maalox). Do not attempt to neutralize an acid burn with an alkaline agent (ie, baking soda/powder); this may cause further tissue damage. If the patient is hypocalcemic, as q uantified by lab analysis, emergency department treatment may include IO mL of a 10% calcium gluconate solution IV. In the prehospital setting, significant cutaneous exposure may be treated with S mL of 10% calcium chloride mixed in 500 mL of normal sali ne for IV infusion. Follow your local protocols.

A 55-year-old male who is an alcoholic stopped drinking 4 days ago. He is confused and restless. His skin is hot to the touch, he is diaphoretic, and his hands are shaking. His BP is 180/90 mm Hg, pulse is 120 beats/min, respirations are 24 breaths/min, and oxygen saturation is 98%. What should you do? A) Assess his blood glucose and establish vascular access B) Establish vascular access and give a 20 mL/kg crystalloid bolus C) Establish va c ular acce s and give 5 mg midazolam IV for sedation D) Assess his blood glucose and give 5 mg diazepam IM to prevent seizures

A) Assess his blood glucose and establish vascular access Feedback: A potentially fatal complication of alcohol withdrawal is delirium tremens (DTs). Symptoms usually start 48 to 72 hours a fter the last alcohol intake (longer in some cases). Mortality rates as high as 15% have been reported. Signs include confusion, tremors, restlessness, fever, diaphoresis, frightening hallucinations, autonomic hyperactivity (eg, hypertension and tachycardia), and in some cases, seizures. If the patient i s severely dehydrated, hypoten ion may be present. Treatment is aimed at protecting the patient from injury and supporting the ABCs. Hallucinations associated with DTs often make for an agitated or combative patient; try to keep the patient calm and oriented, and provide emotional support and reassurance. Give oxygen (if indicated by pulse oximetry), establish IV access (a BP of 180/90 mm Hg does not warrant fluid boluses), and transport. Assess the blood glucose level of any patient with an altered mental status and administer IV dextrose if hypoglycemia is present. Some patients with DTs may develop cardiac dysrhyd1mias, so ECG monitoring is essential. Benzodiazepines (eg, Valium, Ativan) are not used to prevent seizures; they are used to termi nate seizures if they occur. Sedation of a confused patient is not advisable; it may mask other neurologic signs.

Following long-term antibiotic therapy, an older man presents with watery diarrhea, nausea, anorexia, and abdominal pain. His vital signs are stable, although he is slightly tachycardic . What should you suspect? A) Clostridioides difficile B) Vancomycin-resistant E11terococcus C) Methicillin-resistant Staphylococcus aureus D) Carbapenem- resistant Enterobacteriaceae

A) Clostridioides difficile Feedback: Closnidioides difficile (commonly referred to as C diff) is not a multidrug resistant organism but is being treated like one. If can occur after antibiotic treatment because some antibiotics destroy the norrnal bacteria in the intestine, and the C diff organisms take over. Infections with C diff were generally related to a stay in a health care facility, but now are mainly found in the community setting. C diff is now most prevalent in the community setting because of issues with antibiotic prescribing. The spore-forming bacteria C diff produces two endotoxin that cause watery diarrhea, the chief symptom of infection. Infection with C diff causes frequent watery, green, foul-smelling diarrhea; nausea and vomiting; fever; loss of appetite; and abdominal discomfort. one of the other conditions listed present with the symptoms the patient is experienc ing.

A 39-year-old woman is confused, is experiencing hallucinations, and is repetitively smacking her lips. What should you s uspect? A) Complex partial seizure B) Simple partial seizure C) Focal motor seizure D) Generalized seizure

A) Complex partial seizure Feedback: Seizures are classified as being generalized or partial. Your patient's presentation is consistent with a complex partial seizure. Partial seizures affect a limi ted part of the bra in and are further divided into simple partial and complex partial. Simple partial seizures i nvolve movement (frontal lobe) or sensations (parietal lobe) to one part of the body. A focal motor seizure is a simple partial seizure with localized motor activity. There may be spasm or clonus Uerking) of one muscle or muscle group, which may remain localized or may spread to adjacent muscles (Jacksonian march). Complex partial seizures involve changes in level of consciousness. The patient can become confused, lose alertness, experience hallucinations, or may be unable to speak. Automatisms, uch as lip smacking, chewing, or swallowing, may occ u r with complex partial seizures. Generalized seizures affect the entire brain. Tonic-clonic seizures (full body jerking movements), absence seizures (freezing or staring), and pseudoseizures (tonic- clonic, but caused by a psychiatric mechanism) are examples of generalized seizures.

What are the therapeutic effects of epinephrine when administered to a patient with anaphylaxis? A) Constricts the blood vessels and dilates the bronchioles B) Decreases the heart rate, which improves cardiac output C) Blocks histamine receptors and stops the allergic reaction D) Causes fluid to shift back into the cell, which reduces edema

A) Constricts the blood vessels and dilates the bronchioles Feedback: The life-threatening effects or anaphylactic shock are vasodilation (causes hypotension) and bronchoconst riction (impairs breathing). Epinephrine stimulates alpha and beta receptors; as such, it constricts the blood vessels (alpha-I stimulation) and dilates the bronchioles (beta-2 stimulation). Through beta-I stimulation, epinephrine increases the heart rate. The therapeutic effects of epinephrine during anaphylactic shock are vasoconstriction (increases blood pressure) and bronchodilation (improves breathing). Diphenhydramine (Benadryl) is given as an adjunct to epinephrine in anaphylactic shock; it blocks histamine receptors and stops the allergic reaction.

A 39-year-old woman has been taking corticosteroid medications for a long time. She complains of chronic weakness and fatigue and states that she has gained a lot of weight, mainly in her abdomen. Assessment reveals that her face is swollen, and she has multiple skin tears on her arms and legs. What should you suspect? A) Cushing disease B) Addisonian crisis C) Addison disease D) Adrenal insufficiency

A) Cushing disease Feedback: Hyperad renal ism, or Cushing disease (also called Cushing syndrome), is the clinical condition caused by long-standing exposure to excessive circulating serum levels of glucocorticoids, particularly cortisol, because of overproduction in the adrenal cortex. It is more common in women, especially those aged 20 to 50 years. Cushing disease (or syndrome) can be brought on by tumors of the adrenal or pituitary gland, or by long-term corticosteroid use. Patients wid1 Cushing disease have a distinct appearance characterized by obesity, a moon face, and other cardi nal features. Signs and symptoms that tend to accompany this disorder include chronic weakness; increased body and facial hair; a full, pu ffy face; a fatty "buffalo hump" at the back of the neck; central body obesi ty; thin, fragile skin (making the patient prone to skin tears); atrophied proximal muscles; and purple striae (stretch marks) on the abdomen, buttocks, breasts, or arms. Primary adrenal insufficiency. also called Addison disease, occurs when the adrenal cortex fails to produce enough cortisol; in effect, it is the opposite of Cushing disease. Some patients may experience an Addisonian crisis, or acute adrenal insufficiency. Despite its name, one need not have Addison disease to be prone to an Addisonian crisis. The most common cause of Addisonian crisis i s abrupt discontinuation of steroid therapy after prolonged use. It may also occur when a patient fails to receive an adjusted dosage d u ring times of stress, such as during illness or after major surgery or trauma.

A man is unresponsive and has cool, clammy, pale skin. His wife states that he has diabetes and that he wears an insulin pump . His respirations are rapid and shallow and his blood glucose level reads 22 mg/dL. What should you do? A) Deactivate his insulin pump B) Administer 1 mg glucagon IM C) Administer 25 g of 50% dextrose D) intubate his trachea immediately

A) Deactivate his insulin pump Feedback: Insulin pumps are becoming prolific among patients with diabetes, and the paramedic should have a worki ng knowledge of the different types of pumps that are used (they are quite simple to navigate). If a symptomatic patient presents with a documented hypoglycemia, the paramedic should take action to ensure a patent airway and adequate ventilation. If needed, insert a simple airway adj unct and assist ventilations with a bag-mask device; immediate intubation is not a priority with this patient. The next priority would be to deactivate the patient's insulin pump; otherwise, it may continue to deliver insulin, which would further lower the patient's blood glucose level. Establ ish vascular access and administer 12.5 to 25 g of 50% dextrose. If vascular access is not possible, administer glucagon IM.

A woman develops hives on her chest and arms after being stung by a scorpion. She is conscious and alert and tells you that it is difficult to swallow. Her heart rate is 110 beats/min and her oxygen saturation is 94%. Which of her signs o r symptoms is the MOST concerning? A) Difficulty swallowing B) Oxygen saturation of 94% C) Hives on her chest and arms D) Heart rate of 110 beats/min

A) Difficulty swallowing Feedback: The patient is clearly experiencing an allergic reaction, as evidence by the hives (urticaria) on her chest and arms. Of the signs and symptoms listed, however, the presence of difficulty swallowing (dysphagia) should concern the paramedic the most; this suggests angioedema, which could threaten her airway. Her oxygen saturation is low normal, but still within a normal range; this should be closely monitored, however.

An IV has been established on a woman who complains of abdominal pain. During transport, she begins complaining of chills, a backache, and nausea. What should you do? A) Discontinue the IV fluid B) Give a 250-mL fluid bolus C) Administer 4 mg ondansetron D) Administer 0.3 mg epinephrine

A) Discontinue the IV fluid Feedback: The patient's signs and symptoms indicate a pyrogenic reaction. A pyrogenic reaction occurs when fever-causing agents {pyrogens) in the IV fluid produce fever; chills, nausea, and back pain are common. The paramedic should discontinue the IV fluid immediately; disconnect the IV tubing and attach a saline lock. A pyrogenic reaction is not an allergic reaction; therefore, epinephrine is not indicated. I f necessary, administer an antiemetic; 4 mg ondansetron (Zofran) or 12.5 to 25 mg promethazine (Phenergan), depending on your local protocols.

A woman complains of chills, muscle aches, and a headache that began a few days ago. Her blood pressure is 130/72 mm Hg, pulse is 118 beats/min and stro ng, and respirations are 16 breaths/min and unlabored. S he is conscious and alert and denies any past medical history. What add itional finding would you expect to encounter? A) Fever B) Petechiae C) Wheezing D) Hypoglycemia

A) Fever Feedback: This patient's clinical presentation is consistent with in fection with the influenza virus. Chills are generally an indication of fever, and febrile patients are typically tachycardic. In addition to fever and chills, common signs and symptoms of the nu include muscle aches, anorexia, headache, and malaise. These symptoms are often followed by an upper respiratory infection and cough that may last for up to 7 days. Elderly and immunocompromised patients are at greater risk for secondary bacterial infections (eg, pneumonia), in which case localized wheezing may be observed. Because of her mental status and absence of any significant medical problems, hypoglycemia is unlikely, ald1ough it would not be unreasonable to assess her blood glucose level. A rash (ie, purpura, petechiae) is common in patients with severe sepsis; although this patient is ill, she is not septic.

A woman has a rash and severe itching to her trunk and arms that began a day after she started taking penicillin. Her breath sounds are clear to auscultation bilaterally, she denies shortness of breath, her oxygen saturation is 96%, and her BP is 114/64 mm Hg. What should you do? A) Give 25 mg diphenhydramine IM B) Give 0.3 to 0.5 mg epinephrine I M C ) Apply warm compres es t o relieve t h e itching D) Give high-flow oxygen with a nonrebreathing mask

A) Give 25 mg diphenhydramine IM Feedback: The patient is experiencing an allergic reaction; however, there are no signs of anaphylactic shock. She is breathing normally, she has clear breath sounds, and her blood pressure is adequate. Epinephrine is indicated for pati ents with anaphylactic shock, as evidenced by respiratory compromise and/or hemodynamic instability. The most appropriate treatment for this patient would be diphenhydramine (Benadryl) in a dose of 25 to 50 mg IM or IV. Methylprednisolone (Solu-Medrol), in a dose of 125 mg, would also be reasonable. Supplemental oxygen is not indicated for this patient; her oxygen saturation is 96%. Warm compresses would l ikely cause the patient to itch more because heat is a vasodilator. Transport the patient and monitor her closely e n route.

A woman p resents with abdominal cramping, nausea, vomiting, and diarrhea. She is conscious and alert and is lightheaded. BP is 110/70 mm Hg, pulse is 120 beats/min, and respirations are 18 breaths/min. Her temperature is 97.7° F orally and her oxygen saturation is 95%. What should you do? A) Give a 500-m L fluid bolus and 4 mg ondansetron B) Give oxygen by nasal cannula and 2 mg morphine C) Give 200 mL of cold saline and 1 mcg/kg fentanyl D) Give 25 mg diphenhydramine and 50 mg ketamine

A) Give a 500-m L fluid bolus and 4 mg ondansetron Feedback: The patient likely has gastroente1itis, inflammation or infection of the gastrointestinal (GI) tract. Secondary to vomiting and diarrhea, and as evidenced by her symptoms (lightheadedness and tachycardia), she is clearly dehydrated. An absence of fever suggests a viral cause. The norovirus (Norwalk virus) causes most cases of viral gast roenteritis in adults. Treatment includes fluid rehydration and transport. Establish IV access, give a 500-mL fluid bolus (not warmed or cooled), and reassess the patient. Give additional fluid boluse as needed. Nausea and vomiting are created with an anciemetic such as promechazine (Phenergan), 12.5 co 25 mg, or ondansetron (Zofran), 4 mg. Oxygen is not indicated for this patient because she is not hypoxemic. Analgesia is usually not necessary for patients with gastroenteritis. Furthermore, morphine often causes nausea and vomiting, an undesirable side effect in an already dehydrated patient. Diphenhydramine (Benadryl), an antihistamine, is used to treat allergic reactions; although it does possess a ntiemetic properties, it is not the fi rst choice for patients with nausea and vomiting.

You are assessing a you ng female and are trying to determine if she experienced a seizure or a syncopal episode. Which of the following findings is MOST consistent with a seizure? A) Her heart rate is 120 beats/min. B) The episode occurred while she was standing. C) A bottle of lorazepam is found in her purse. D) She complains of weakness and lightheadedness.

A) Her heart rate is 120 beats/min. Feedback: It can be difficult to detennine whether a patient experienced a seizure or a syncopal episode (fainting). Therefore, it is important to ascertain what happened before and after the episode. Tachycardia is a common finding following a seizure, when the patient is postictal; it is caused by a sympathetic nervous system discharge that occurred during the seizure. Bradycardia often causes syncope secondary to an increase in vagal tone. Seizures occur regardless of the patient's position, whereas syncope typically occurs while the patient i standing. Most patient with a history of seizures take medications that prevent seizures, such as phenytoin (Dilantin), valproic acid (Depakote), and carbamazepine (Tegretol), among others. Although benzodiazepines (ie, diazepam [Valium], lorazepam [Ativan]) are the most common class of drugs used to terminate seizures, they are more commonly prescribed for anxiety; they are rarely prescribed to prevent seizures.

Tro usseau sign may be encountered in patients with which of the following conditions? A) Hypocalcemia B) Hyperkalemia C) Type II diabetes D) Hyperthyroidism

A) Hypocalcemia Feedback: You should be familiar with Trousseau sign and Chvostek sign, both of which will help you detect muscular irritabili ty caused by hypocalcemia. To obtain a positive Trousseau sign, place a BP cuff around the arm, inflate i t to 30 mm Hg above the systolic BP, and hold it in place for 3 minutes. This will induce carpopedal spasm. You can elicit a positive Chvostek sign by tapp ing the facial nerve against the mandible just anterior to the ear, which produces a spasm of the facial muscles. This sign, however, is not as sensitive as Trousseau sign. None of the other conditions listed is associated with Trousseau or Chvostek sign.

A man ingested a bottle of extra strength acetaminophen . He presents with malaise and palpable tenderness to the right upper quadrant of his abdomen. He is conscious and alert, his BP is 136/66 mm Hg, his pulse is 100 beats/min and regular, his respirations are 16 breaths/min, and his oxygen saturation is 95%. Which of the following treatments is indicated for him? A) IV saline lock B) High-flow oxygen C) 2 g/kg of activated charcoal D) 1 mEq/kg of sodium bicarbonate

A) IV saline lock Feedback : Acetaminophen (APAP), the active ingredient in Tylenol, is rapidly absorbed from the GI tract, producing peak serum levels in 30 to 120 minutes. Your patient's signs and symptoms are consistent with Stage I I toxicity (ie, malaise, RUQ pain). He is conscious and alert, his breathing is adequate, his vital sign are stable, and his SpO2 is greater than 94% on room air. Therefore, treatment should incl ude an IV, either as a saline lock or otherwise at a keep-vein -open (KVO) rate; cardiac monitoring; and transport. His BP does not warrant fluid boluses. Activated cha rcoal would be of no benefit; the drug is no longer in the stomach, it is systemic. Sodium bicarbonate, if indicated, should be guided by a rterial blood gas analysis. The antidote for APAP toxicity is acetylcysteine (Acetadote); however, it i only given at the hospital.

Which of the following statements regarding sickle cell disease is correct? A) In sickle cell disease, misshapen red blood cells can lodge in the spleen, causing it to swell and rupture. B) When the sickle cell gene is inherited from both parents, the patient will probably not develop the disease. C) The red blood cells in sickle cell disease have a rounded appearance, rather than the normal oblong appearance. D) Despite their altered shape, the red blood cells of a person with sickle cell disease are effective carriers of oxygen.

A) In sickle cell disease, misshapen red blood cells can lodge in the spleen, causing it to swell and rupture. Feedback: Sickle cell disease is the most commonly inherited blood disorder. Although it primarily affects African-American, Puerto Rican, and European populations, it can occur in anyone. Sickle cell disease starts with a gene defect of the adult-type hemoglobin (HbA). This mutated gene can be inherited from one parent (HbS) or both parents (HbSS). When the gene is inherited from both parents, there is a high probability that the patient will be prone to sickle cells (that is, the person actually has the disease) or the sickle cell trait (the person is a carrier of the mutated gene). The defective red blood cells (RBCs) are misshapen; affected cells have an oblong (sickle) shape instead of a mooth, round shape. This shape makes the RBCs poor carriers of oxygen, which means that a person with sickle cell disease is highly susceptible LO hypoxia. The odd shape may also cause RBCs Lo lodge in small blood vessels (thrombotic cri is) or in the spleen (splenic sequest ration crisis), causing the organ LO swell and rupture, which can cause death. Sickle cell disease may also lead to other crises, such as aplastic crisis and hemolytic crisis. I n aplastic crisis, RBC production temporarily stops. In hemolytic crisis, the RBCs break down quickly. In acute crises, patients often have severe pain, which will require aggressive pain management. They may also experience frequent infections, which can lead to sepsis and death.

A 24-year-old woman presents with a headache , fever, and photophobia. She is disoriented and slow to answer your q uestions. Your assessme nt reveals a positive Brudzinski sign. What should you suspect? A) Meningitis B) Encephalitis C) Epidural hemorrhage D) Subdural hemorrhage

A) Meningitis Feedback: Give the patient's clinical presentation and your assessment findings, you should suspect meningitis. Key findings of meningitis include fever, altered mental status, and meningismus (a triad of nuchal rigidity, photophobia, and headache). Brudzinski sign is caused by meningeal irri tation that results in an involuntary Oexion of the knees when the head is flexed toward the chest; it is also a sign of meningitis. Encephalitis i general inflammation of the brain that cause focal or diffuse brain dy function. Because the meninges are not affected, you would not expect to encounter Brudzinski sign with encephalitis. It should be noted, however, that some patients can have both en cephalitis and men ingitis. lntracranial hemorrhage could explain some of the patient's symptoms; however, the presence of fever and positive Brudzinski sign make these less likely.

A patie nt, who is conscious and has a p ulse, has a core body temperature of 92.8° F. Which of the following treatments is correct? A) Passive rewarming only B) Active external rewarming C) Prophylactic antidysrhythmic therapy D) 20-mL/kg bolus of warm normal saline

A) Passive rewarming only Feedback: A patient with mild hypothermia (CBT of 90°F to 95°F [32°C to 35°C]) should be treated with passive rewarmi ng, which involves removing wet clothing, drying the patient off, applying warm blankets, and allowing the patient's CBT to rise naturally. If the patient has moderate hypothermia (CBT of 82°F to 90° F [28°C to 32°C]), active external rewarming should be performed because passive rewarming alone is generally not sufficient. This approach involves the use of several means to directly warm the patient's skin, including heating blankets and forced hot air. Commercial warming devices that u e special blankets and a heated fan unit can warm patients at a rate of up to 4.3° F (2.9°C) per hour, which is much faster than warm blankets. Severe hypothe,mia (C BT of less tha n 82°F [28°C]) is treated with active internal rewarn1ing, which is accomplished with the use of warmed IV fluids, warm humified oxygen, body cavity lavage, extracorporeal rewarming, and esophageal rewarming tubes. In most cases. active internal rewam1ing will rake place at the hospital. Prophylactic antidysrhythmic therapy is not indicated for hypothermic patients.

Which of the following complications would likely occur if a patient missed several dialysis treatments? A) Pulmonary edema B) Bacterial infection C) Severe dehydration D) Marked hypokalemia

A) Pulmonary edema Feedback: Dialysis filters waste products from the blood. If the patient misses their dialysis treatment, these waste products accumulate in the blood. A n u mber of negative effects can occur, including overhydration, pulmonary edema, and hyperkalemia, among others. Hypokalemia and dehydration can occur with aggressive dialysis.

A woman complains of a burning sensation in her abdomen but is unable to describe exactly where it hurts. How would you describe this type of pain? A) Visceral B) Parietal C) Somatic D) Referred

A) Visceral Feedback: Visceral pain occurs when hollow abdo minal organs contract too forcefully or are distended. This type or pain is difficult to localize and is often described as burning, aching, gnawing, or cramping. The pain is usually felt superficially. Somatic pain is felt deeply and is usually well localized. This type of pain is caused by irritation of or injury to tissue, causing activation of peripheral nerve tracts. Parietal pain is a steady, aching pain that is easier to localize than visceral pain. It is caused by inflammation of the peritoneum (peritonitis) due to bleeding or infection. Referred pain is pain that originates in the abdomen and causes the perception of pain in distant locations. It i attributable to similar path from the peripheral nerves or the abdomen and those in the distant location. An example of referred pain is Kehr sign, which is pain in the t ip of die shoulder caused by blood or other i rritants in the peritoneal cavity when a person is lying down with legs elevated. Kehr sign is considered a classic symptom of a ruptured spleen.

A 29 -year-old man is agitated and refuses to give you any information because his roommate told him not to trust anyone. His small apartment clearly indicates that he is the sole occupant. Your partner finds a prescription for Zyprexa, which is prescribed to the patient. What should you suspect? A) Schizophrenia B) Bipolar disorder C) Trichotillomania D) Obsessive-compulsive d isorder

A) Schizophrenia Feedback: Schizophrenia is a psych iatric illness marked by impairments in the perception or expression of reality. Signs include auditory hallucinations (eg, the "roommate"), paranoid or bizarre delusions (eg, you are not to be trusted), and disorganized speech and thinking. Antipsychotics such as olanzapine (Zyprexa), risperidone (Risperdal), and clozapine (Clozaril) are commonly prescribed to creaL schizophrenia. The patient's presentation is not consistent with bipolar disorder. Bipolar disorder is a biphasic psychiatric condition characLerized by alternations of depres ion and mania. Manic patients are excessively elated, overly talkative, and have accelerated speech. Medications used to treaL bipolar disorder include lith ium, q uetiapine fumarate (Seroquel), and aripiprazole (Abili fy). Trichotillomania, the recurrent pulling out of one's own hair, is an impulse control disorder. Nothing in the scenario suggests this condition. Obsessive-compulsive disorder (OCD) is a psychiatric condition characterized by intrusive and unwanted thoughts or worries (obsessions), which the patient attempts to alleviate or eliminate by perforn1ing ritualistic acts (compulsions). Selective serotonin reuptake inhibitor (SSRI) drugs, such as 0uoxetine (Prozac), senraline (Zoloft), and paroxetine (Paxil), are commonly prescribed to patients with OCD. This patient's behavior is not consistent with OCD.

Which of the following observations is the MOST suggestive of a person's potential to become violent? A) Sitting, clenched fists, erratic speech B) Standing facing you, ar ms crossed C) Large body size, sitting, flat affect D) Sitting, crying, unable to recall birthday

A) Sitting, clenched fists, erratic speech Feedback: The potential for violence exists on every call the paramedic responds to. People respond to stress differently; anyone involved in an incident-the patient, Family members, or bystanders-can become angry or violent. In patients with certain preexisting mental illnesses (eg, intermittent explosive disorder, sch izophrenia), the potential for violence is clearly higher, and the paramedic must be able lo recognize certain behaviors that indicate this. A "fighting stance" position-the patient is not face-co- face with you, they standing sideways-is an engaging position, indicating that the patient could lunge at you at any moment. Other warning sign include a tense appearance; clenched fists; pacing; agitation; and erratic, loud, or obscene speech. Crying and a flat affect are indicative of depression; however, this does not mean that the depressed patient cannot suddenly turn violent. There is absolutely no correlation between a patient's physical size and their potential for violence.

A woman was bitten on the ankle by a rattlesnake. She is conscious and alert, but anxious. Her BP is 144/72 mm Hg, her pulse is 116 beats/min and strong, and her respirations are 16 breaths/min and unlabored. What should you do? A) Splint the entire lower extremity B) Apply an icepack directly to the site C) Give 1 g of calcium chloride IV push D) Elevate the extremity above the heart

A) Splint the entire lower extremity Feedback: Treatment for a pit viper (rattlesnake, copperhead, cottonmouth) bite includes placing the patient in a comfortable position, admin istering oxygen (if hypoxemic), splinting the entire affected extremity and keeping at below the level of the heart, and prompt transport. Alert the receiving facility early and confirm that they have an antidote (Crofab) available. Monitor the patient’s vital signs, ECG, oxygen saturation, and end-tidal CO2. Establish IV access, but do not administer fluid boluses unless die patient is hypotensive. Calcium chloride is not indicated for snakebite ; in some cases, it (or diazepam) may be given to pat ients b itten by a black widow spider. Do I OT apply ice (in any form) for a snakebite; doing so causes localized vasoconstriction. If envenomation occurred. this vasoconstriction could push the venom further into the circulation & nbsp

You wo uld expect to encounter trismus with which of the following condit ions? A) Tetanus B) Rabies C) Mumps D) Meningitis

A) Tetanus Feedback: The most common presenting manifestation in patients with teta nus is trismus, spasm of the jaw muscles that causes difficulty opening the mouth-hence the term "lockjaw." Tetanus is a potentially fatal disease of the central nervous system caused by the bacterium Clostridium tetani . If spores of C tetani enter the circulatory system, they multiply and produce a neurotoxin called tetanospasmin, which attacks the nerves that regulate muscular activity. /Is tetanospasmin circulates more widely, it interferes with the normal activity of nerves throughout the body, resulting in generalized muscle spasms. Any open wound that is contaminated with soil, saliva, or feces-especially if not properly cleaned-and puncture wounds from nonsteri!e items such as nails or needles, are at h igh risk of being infected wi th C tetani. Rabies, mump , and meningitis-all of which can be fatal-are typically not associated with t rismus.

Which of the following occurs when the blood gl ucose l evel falls, such as during strenuous exercise? A) The pancreas secretes glucagon, which stimulates the liver to convert glycogen to glucose. B) The pancreatic beta cells cease insulin production altogether until the blood sugar level increases. C) The cells metabolize fat, which produces a weaker form of glucose that the body can still utilize. D) The body's cells release their glucose stores into the bloodstream to maintain homeostasis.

A) The pancreas secretes glucagon, which stimulates the liver to convert glycogen to glucose. Feedback: When the body's blood glucose level falls, the alpha cells of the pancreas secrete the hormone glucagon, which stimulates the liver to convert glycogen to glucose, a process called glycogenolysis. The liver then releases glucose into the bloodstream, thereby increasing circulating glucose levels. The body's cells do not release glucose into the bloodstream; l ike the brain, the cells need glucose to survive. Cellular fat metabolism results in the formation of ketoacids, which occurs when insulin supplies are insufficient or absent and glucose accumulates in the blood because it is unable to enter the cell (eg, hyperglycemic ketoacidosis).

Which of the following statements regarding hypoglycemia is correct? A) With severe hypoglycemia, the patient's response to treatment may be delayed or short-lived. B) Most diabetic patients develop symptoms when their blood glucose level falls below 90 mg/dL. C) The skin of a hypoglycemic patient is typically warm and dry secondary to severe dehydration. D) Hypoglycemia typically occurs within 4 to 6 hours after inadvertently taking too much insulin.

A) With severe hypoglycemia, the patient's response to treatment may be delayed or short-lived. Feedback: Administering 12.5 to 25 g (25 to 50 mL) of 50% dextrose (D50) is usually sufficient to restore adequate circulating blood glucose levels and improve the patient's cond ition. However, if the hypoglycemic event is severe or prolonged, the patient's response to treatment may be short-lived and/or delayed, thus requiring additional dosing. Some agencies use 10% dextrose (DI0) to treat hypoglycemia in order to avoid an acute spike in blood glucose that can be seen with D50. It is important to assess th e patient's blood glucose level (BGL) and mental status after administering dextrose; doing so will help determine if additional dextrose is needed. annal serum glucose levels range from 70 to 120 mg/dL. Hypoglycemia following an inadvertent overdose of insuli n typically develops within seconds t o minutes. T h e point at which a hypoglycemic patient becomes symptomatic varies. Some patients may develop symptoms w h e n their BGL falls below 70 or 80 mg/dL; others may not develop symptoms until their BGL falls below 40 or 50 mg/dL. Diabetic patients with severe hypoglycemia typically have cool, clammy skin. Patients w ith severe hyperglycemia typically present with warm, dry skin; this is a manifestation of dehydration secondary to the osmotic diuresis associated with hyperglycemia.

Treatment for a black widow spider bite includes which of the following? A) 10 mL calcium chloride B) 2.5 to 10 mg diazepam C) 2 mg/kg 10% calcium gluconate D) Antivenin derived from horse serum

B) 2.5 to 10 mg diazepam Feedback: In addition to ainvay, breathing, and circulatory support, opioids and muscle relaxants/sedatives are the prehospital mainstays of therapy when treating a black widow spider bite. To Lreat the muscle spasms, give 2.5 to 10 mg of d iazepam (Valium); lorazepam (Ativan) may also be used. Some EMS system protocols may call for 0.1 to 0.2 mg/kg of calcium gluconate. Narcotics, such as fentanyl or morphine, can be used for pain relief. Calcium chloride is nol effective in the treaLment of a black widow spider bite. An antivenin is available for treatment of black widow spider bites; however, Lhis is not a prehospital intervention. Furthermore, this antivenin i typically re erved for the young and old who have severe envenomation; it also has a high incidence of allergic reactions inherent in all equine-derived (horse se rum) antivenin.

Following ingestion of a toxic dose of acetaminophen, right upper quadrant abdominal pain typically begins within what time frame? A) 12 to 24 hours B) 24 to 72 hours C) 72 to 96 hours D) 4 to 14 days

B) 24 to 72 hours Feedback: Acetaminophen, the active ingredient in Tylenol, can cause liver failure and dead1 if a toxic dose is ingested. A unique aspect of acetami nophen toxicity is that i ts signs and symptoms appear in four distinct stages. Stage I (less than 24 hours) sym ptoms include nausea, vomiting, anorexia, pallor, and malaise. Stage II (24 to 72 hours) symptoms include right upper quadrant (RUQ) abdominal pain and abdominal tenderness to palpation. Stage III (72 to 96 hours) symptoms include metabolic acidosis, renal failure, coagulopathies, and recurring GI symptoms. During Stage IV (4 to 14 days [or longer]), recovery slowly begins or liver failure progresses and the patient dies. The antidote for acetaminophen toxic i ty is acetylcysteine (Acetadote); ideally, it should be given within 8 hours following ingestion.

Which of the following patients is at greatest risk for hypothermia? A) 45-year-old man with hyperglycemia B) 55-year-old woman with hypothyroidism C) 60-year-old woman with Cushing syndrome D) 65 -year-old man with coronary artery disease

B) 55-year-old woman with hypothyroidism Feedback: Hypod1yroidism is a condition in which the thyroid gland produces too little T3 (triiodothyronine) and T4 {thyroxine), resulting in a decrease in the metabolic rate. Any time the metabolic rate decreases, heat energy production is reduced; therefore, the patient is prone to hypothermia. Cushing syndrome is caused by excessive cortisol production by the adrenal glands or by excessive use of cortisol or other similar glucocorticoid hormones (ie, prednisone, hydrocortisone, methylprednisolone). This increase in cortisol would cause an increase in the metabolic rate. Patient's with Cushing syndrome are not at risk for hypothermia, nor are patients with hyperglycemia or coronary artery disease.

Which of the following assessment findings is indicative of peritoneal irritation? A) Pain that increases when the patient is placed on their side B) A decrease in pain when drawing the knees into the abdomen C) A relief of pain when the patient moves around frequently D) Pain that is referred to the shoulder, neck, or scapulae

B) A decrease in pain when drawing the knees into the abdomen Feedback: The classic position for the patient with peritoneal inflammation or irritation is recumbent with the knees drawn up into the chest (fetal position). This position takes stress off of the abdominal musculature, thereby affording some pain relief. Even small bumps in the road can cause intense pain in patients with peritonitis.

Which of the following assessment findings helps dist inguish envenomation d ue to a black widow spide r bite from an acute abdominal condition? A) Dizziness, nausea, vomiting, and diaphoresis B) Abdominal rigidity without palpable tenderness C) Hematemesis, dysphagia, and significant hypotension D) Fever, palpable abdominal tenderness, and tachycardia

B) Abdominal rigidity without palpable tenderness Feedback: The venom of a black widow spider contains a neurotoxin that causes severe muscles spasms and intense pain - especially of the abdomen. Unlike an acute abdominal condition, however, the patient's abdomen is typically not tender to palpation. Dizziness, nausea, vomiting, and diaphoresis are common in patients with an acute abdominal condition and black widow spider envenomation. Fever and tachycardia in conjunction with abdominal tenderness is consistent with acute abdominal conditions such as peritonitis and appendicitis, not envenomation from a black widow spider. Hematemesis, dysphagia, and hypotension are common assessment findings in patients with ruptured esophageal varices.

A 60-year-old male with chronic alcoholism presents with an acute onset of hematemesis. His BP is 80/40 mm Hg, pulse is 130 beats/min and weak, and respirations are 28 breaths/min and shallow. What should you suspect? A) Erosion of the large esophageal blood vessels caused by the toxic effects of alcohol B) Abnormally enlarged esophageal veins secondary to impaired blood flow to the liver C) Severe bleeding from gastric ulcers that formed due to the excessive intake of alcohol D) Acute rupture of an esophageal artery secondary to repeated episodes of vomiting

B) Abnormally enlarged esophageal veins secondary to impaired blood flow to the liver Feedback: This patient likely has ruptured esophageal varices. Esophageal varices are a complication of portal hypertension (increased pressure in the portal ve in [the vein that carries blood from the intest ines to the liver]) due to liver disease. Liver disease (ie, hepatitis, cirrhosis) is common in patients with chronic alcoholism. /Is blood flow to the diseased liver is reduced, blood backs up into the smaller, more fragile blood vessels i n the lower part of the esophagus. As a result, these esophageal vessels become abnormally enlarged (varices). Esophageal varices are generally asymptomatic unless they rupture, in which case life-th reatening hemorrhage can occur. Other complications associated with chronic alcoholism include impaired blood-clott ing mech anisms, hypoglycemia, and gastritis due to the irritant effect of alcohol on the gastric lining. Repeated episodes of vomiting could rupture an esophageal vessel and cause hematemesis; however, there is no indication that this patient has had repeated episodes of vomiting.

Which of the following findings is the MOST clinically significant when assessing a patient with a headache? A) Nausea or vomiting B) An unsteady gait C) Retroorbital pressure D) BP of lS0/82 mm Hg

B) An unsteady gait Feedback: Most headaches are benign; they are usually not a symptom of a serious underlying problem. Migraine headaches, cluster headaches, tension headaches, and sinus headaches - albeit painful and bothersome - are not life-threatening, and are responsible for the majority of headaches that patients experience. Nausea and vomiting are common secondary complaints in patients experiencing migraine or cluster headaches. Retroorbital pressure - pressure behind the eyes - is very common in patients experiencing a sinus headache. Hypertension can cause a severe headache; however, the patient's systolic BP is often as high as 180 to 200 mm Hg. Furthermore, hypertension is often due to a sympad1etic nervous system d ischarge in response Lo the pain of the headache itself. In some patients, a severe headache is caused by a serious un derlying problem, such as an intracranial lesion (eg, brain tumor), cerebral abscess, or cerebral aneurysm. In these cases, the expanding lesion or hematoma can cause damage to or put pressure on certain parts of the brain, resulting in abnormal neurologic signs. An unsteady or staggering gait is not a common clinical findi ng in patients with a benign headache; you should suspect a serious underlying problem if it i s observed.

An elderly man is not acting right, according to his daughter. He becomes agitated when you attempt to assess him and tells you that you are not taking him anywhere. What should you do? A) Gently restrain him and transport him to the hospital B) Calmly talk to him and try to obtain a glucose reading C) Administer 5 mg diazepam IM to calm and sedate him D) Start an IV line and administer 12.5 gm of 50% dextrose

B) Calmly talk to him and try to obtain a glucose reading Feedback: Any patient with an altered mental status should be ruled out for hypoglycemia by obtaining a blood glucose reading. Administering IV dextrose without assessing his blood glucose level first is not advisable. If he is experiencing an intracranial hemorrhage, dextrose may exacerbate his condition. Conversely, if you discover that he is hypoglycemic, this must be corrected. It is important to calmly approach any patient, regardless of their mental status. The assumption of a psychiatric crisis is not in the best interest of t he patient. Focus on ruling out medical problems first.

Which of the following describes the pathophysiology of multiple sclerosis? A) Dysfunction or damage to the portion of the brain that is responsible for the production of dopamine B) Autoimmune disorder in which the body attacks the myelin sheath of the neurons in the brain and spinal cord C) Genetic disorder in which defective D A causes an error in muscle tissue, such that the malformed muscle cells rupture D) Inflammation of the trigeminal nerve that leads to deterioration of the myelin sheath and causes severe, chronic pain

B) Autoimmune disorder in which the body attacks the myelin sheath of the neurons in the brain and spinal cord Feedback: Multiple sclerosis (MS) is an autoimmune disorder in which the immune system recognizes the protein that makes up the myelin sheath-the protective i nsulation that coats the axons of most nerve cells (neurons) -as being foreign and creates antibodies, called autoantibodies, that destroy it. This leads to areas of scarring that produce symptoms such as muscle weakness; impairment of pain, temperature, and touch senses; pain (moderate to severe); ataxia; tremors; and speech disturbances. Parkinson disease results from damage to or dysfunction of the substantia nigra, the portion of the brain that produces dopamine. Patients with Parkin on disease classically present with fine mu cle tremors. A genetic disorder in which defective DNA causes an error in muscle tissue, such that the malfom1ed muscle cells rupture more easily, is called muscular dystrophy (MD). M D typically presents with progressive muscle weakness, delayed development of muscle motor skills, ptosis (drooping of the upper eyelid), drooli ng, and poor muscle tone. Trigeminal neuralgia, also called tic douloureux, is an inflammation of the t,igeminal nerve (fifth cranial nerve). The trigeminal ne,ve receives sensory information from the face. The usual cause of trigeminal neuralgia is irradiation by an artery lying too close to the nerve. Over time, as the artery changes diameter to meet blood supply needs, the myelin sheath is grated off the ne,ve. With its insulation gone, the nerve may "short circuit," causing severe hock-like or stabbing pain, usually on one side of the face.

Which of the following is a class of lymphocyte that matures in the bone marrow and produces antibodies? A) T cell B) B cell C) Basophil D) Neutrophil

B) B cell Feedback: B cells are a class of lymphocyte that mature in the bone marrow and play an important role in the adaptive immune response. Also known as plasma cells, they produce antibodies (lgA, lgE, lgG, and lgM) that play a role in the immune system. T cells are also a class of lymphocyte; however, they mature in the thymus, not the bone ma rrow. T cells play an important role in escalating the adaptive immune response. There are two types of T cells: helper and cytotoxic/killer. Basophils and neutrophils are leukocytes, not lymphocytes. Basophils play a role in parasitic infections and allergic reactions; they release histamine and heparin. eutrophil comprise approximately 50% of all circulating leukocytes and are typically the first to encounter a foreign microbe from a bacterial, fungal, or viral infection. They can surround the offending agent, ingest it (phagocytosis), and release e nzymes that destroy the organ ism .

What components of the immune system release the body's chemical mediators in anaphylaxis? A) T cells and eosinophils B) Basophils and mast cells C) Histamine and cytokines D) Neutrophils and B cells

B) Basophils and mast cells Feedback: The basophils and mast cells produce the body's chemical mediators. These cells contain granules filled with a host of powerful substances that are ready to be released to fight invading antigens. As long as the body is not invaded by one of the previously identified foreign substances, the granules remain encapsulated in their protective walls and remain inactive. If an antigen invades the body, however, the granules are ejected from the mast cells, a process called degranulation, and the chemical mediators are then released into the surrounding tissue and the bloodstream. Chemical mediators that are released include histamine, eosinophil chemotactic factor, prostaglandin, leukotrienes, platelet-activating factor, serotonin, heparin, chondroitin sulfate, cytokines, and kini ns. These chemical mediators are responsible for the life-threatening effects of anaphylaxis, s uch as hypotension, breathing compromise, and platelet aggregation. Epinephrine is such a vital drug for anaphylaxis because it stops the process of mast cell degranulation; it also reverses the effects of the chemical mediators released via degranulation.

A man was working in his shed when he experienced a sudden, sharp pain in his leg. Assessment of the area reveals a small area of redness, but no visible marks. He is disoriented and complains of intense abdominal pain. What should you suspect? A) Brown recluse spider B) Black widow spider C) Pit viper D) Coral snake

B) Black widow spider Feedback: This is a classic case of a black widow spider bite. The black widow spider typically can be found near wood piles or wood sheds. The patient will usually experience immediate sharp pain at the time of the bite, and then wi thin a short period of time painful muscle spasms will develop in all of t he major muscle groups, especially the abdomen. The black widow spider carries a neurotoxin in its venom, which explains the muscle spasm . If left untreated, central nervous system depression will continue and the patient will experience cardiovascular and respiratory yscem collapse. In contrast to a black widow spider bite, the bite of a brown recluse pider is usually painless, and the patient does not even realize they have been bitten until a red area with a small blister in the center of it appears several hours to a day later. Unlike the black widow spider, the brown recluse spider carries a cytotoxin (necrotoxin) in its venom. Cytotoxins cause tissue and cellular necrosis. Snakebite is unlikely because there are no visible puncture wounds or marks. Even i f he was bitten by a snake, but was not envenomated, you would not expect the symptoms he i presenting with.

A 60-year-old male presents with an acute onset of confusion and slurred speech. His family states that the episode began while he was watching TV, and that he was normal all morning. Which of the following assessments would be the MOST likely to reveal a possible cause of his altered mental status? A) Oxygen saturation B) Blood glucose level C) Palpation of the skull D) Systolic blood pressure

B) Blood glucose level Feedback: Of the assessments listed, the patient's blood glucose level (BGL) would be the most likely to reveal the possible cause of his altered mental status, and hypoglycemia could explain his symptoms. The BGL is a more specific assessment than the others listed. Abnormalities in oxygen saturation and systolic blood pressure can indicate a variety of underlying etiologies. Given the scenario and the onset of symptoms, trauma is unlikely; however, it would still be reasonable co palpate his kull for gro s abnom1alities.

A man has a headache and visual dist urbances that have progressively worsened over the past 3 months. What should yo u sus pect? A) Subdural hematoma B) Cerebral neoplasm C) Epidural hematoma D) Bacterial meningitis

B) Cerebral neoplasm Feedback: The patient's symptoms are consistent with a space-occupying intracranial lesion such as a neoplasm (tumor or growth), which typically presents with a headache, visual disturbances, and other symptoms that progressively worsen over a period of several months. In some patients, a new-onset of seizures may be the only presenting sign of a brain tumor. Subdural hematomas commonly present with symptoms within 12 to 24 hours following head trauma. fin epidural hematoma presents with symptoms immediately following a head injury and causes rapid clinical deterioration. Patients with bacterial meningitis also experience a rapid progression of symptoms.

A 70-year-old woman complains of tinnitus and difficulty concentrating. She tells you that she has consumed six cups of ice over the past hour. What should you suspect? A) Acute le ukemia B) Chronic anemia C) Polycythemia D) Lymphoma

B) Chronic anemia Feedback: Tinnitus (ringing in the ears), a craving for ice, and difficulty concentrating are hallmark findings of chronic anemia. Anemia is defined as a deficiency of red blood cells or hemoglobin. Other findings may include a headache, d izziness, tachycardia, pallor, and shortness of breath. Polycythemia, an overproduction of red blood cells, causes nushing of the skin; it is not commonly associaLed with the clinical signs that this patient i s exhibiLing. Leukemia and lymphoma are types of cancer that commonly manifest with persistent weakness, fever, and swollen lymph nodes (lymphadenopathy).

A man ingested unknown quantities of ibuprofen, aspirin, codeine, and acetaminophen. He is unresponsive, his respirations are 6 breaths/min and shallow, and his heart rate is 50 beats/min. Which of these drugs would cause his clinical presentation? A) Aspirin B) Codeine C) I buprofen D) Acetaminophen

B) Codeine Feedback: Of the drugs listed, codeine is the most likely cause of this patient's clinical presentation. Codeine is a opioid analgesic, and when taken in excess, it can result in central ne,vous system depression (ie, altered mental status, hypoventilation, bradycardia, hypotension). Aspirin toxicity causes metabolic acidosis and would be expected to manifest with hyperventilation, an indication that the respiratory buffer system is attempting to eliminate acids from the body with a compensatory respiratory alkalosis. Ibuprofen, the active ingredient in Advil and Motrin, erodes the gastric lining and can be toxic to the liver; central nervous syste111 depression is not indicative of ibuprofen toxicity. Acetaminophen (/\PAP), the active ingred ient in Tylenol , causes damage to the liver; CNS depression would not be expected with APAP toxicity.

A 65-year-old obese female has experienced a change in behavior over the past several days, according to her husband. She initially seemed forgetful, but today she is confused. Her BP is 90/50 mm Hg, pulse is 50 beats/min, and respirations are 10 breaths/min. Her skin is cold and dry. Her husband is not aware of any medical problems that she has, but remarks about her weight gain over the past few months. What should you suspect? A) Acute dementia with hypothermia B) Decreased thyroid hormone production C) Excess adrenal gland production of cortisol D) Exacerbation of undiagnosed hyperthyroidism

B) Decreased thyroid hormone production Feedback : Myxedema coma is a potentially life-threatening complication of hypothyroidism. Hypothyroidism is the result of decreased thyroid hormone secretion by the thyroid gland. Thyroid hormones are critical for cell metabolism and organ function. Adule hypothyroidism, often called myxedema, causes slowing of the body's metabolic processes. Symptoms are due to a slow metabolic race (fatigue, feeling cold, weight gain, dry skin, sleepiness). Myxedema coma is a n extreme manifestation of hypothyroidism marked by bradycardia, hypotension, and a decreased mental status. Family members may overlook subtle changes, but obvious changes (eg, confusion, psychosis, coma) usually elicit a call to 9-1-1. Myxedema coma usually occurs during the winter in women older than 60 years of age. HypotJ1ermia is common; fever may be absent during an infection. Hyperthyroidism results from excessive thyroid hormone production; it manifests with signs of increased metabolism (eg, tachycardia; weight loss; hot, flushed skin). Dementia is not an acute event; it is a gradual deterioration of cognitive function (eg, Alzheimer's disease). Excess cortisol secretion by the adrenal glands causes Cushing syndrome. Signs and symptoms include weakness, fatigue, depression, hypoglycemia, darkened skin (acanthosis) on the neck, and a moon-face appearance.

Which of the following is a distorted sense of smell, in which the person perceives u npleasant odors that do not exist? A) Anosmia B) Dysosmia C) Hyperosmia D) Presbyosmia

B) Dysosmia Feedback: Loss of smelling sensation can have many different causes, including aging, smoking, allergies, rhinitis, polyps, the nu, medications, and traumatic brain i njury (damage to cranial nerve I [olfactory nerve]). Dysosmia is a distorted sense of smell in which the person perceives unpleasant odors when the odors do not exist. The total loss of sense of smell is called anosmia. Inc reased sensitivity to smell is called hyperosmia. Presbyosmia is a decreased sense of smell caused by normal aging. Loss or smell also affects a person's sense or taste.

A 34-year-old man is in anaphylactic shock. You have administered two doses of epinephrine 0.3 mg via the intramuscular route; however, the patient is still hypote nsive and has labored breathing. What should you do? A) Administer 50 mg of diphenhydramine IM and reassess B) Establish vascular access and begin an epinephrine infusion C) Do uble the dose of epinephrine and give another IM injectio n D) Establish vascular access and give 1 2 5 m g of methylprednisolone

B) Establish vascular access and begin an epinephrine infusion Feedback: Many patients will require more than one dose of epinephrine. If there is no response to intramuscular (IM) epinephrine, an IV epinephrine infusion should be administered (2 to 10 mcg/min) in conjunction with a n IV fluid bolus t o support t h e patient's hemodynamic status as needed. I V or I O fluid boluses are only recommended i f you are unable t o quickly deliver an epinephrine i n fusion and the patient is in a peri-arrest condition. It is important to note that delays in administering epinephrine are considered a major contributing factor to fatalities. Neither diphenhydramine (Benadryl) nor methylprednisolone (Solu-Medrol) wil l benefit the patient with ongoing anaphylaxis; THE lifesaving drug is epinephrine! In severe cardiovascular collapse, the I M route may not be efficient in getting a drug to where it needs to be. Therefore, it would be of minimal to no benefit to double the dose and repeat the IM injection. The intravasc ular route is indicated and preferred for refractory anaphylaxis.

A patient presents with polyuria and a blood glucose reading of 412 mg/dL. Treatment should include which of the following? A) 10 units of insulin via the subcutaneous route B) Fluid rehydration with an isotonic crystalloid C) 25 g of 50% dextrose via the intravenous route D) 40 mg furosemide via the intravenous route

B) Fluid rehydration with an isotonic crystalloid Feedback: Patients with blood glucose levels of 400 mg/dL and polyuria are dangerously close to hyperglycemic ketoacidosis, if not already there. High levels of blood glucose promote an osmotic diuresis, which explains the excessive urination (polyuria); this results in significant dehydration. Prehospital treatment is aimed at rehydrating the patient with an isotonic crystalloid sol ution (ie, normal saline). The patient needs insulin; however, it is rarely given in the prehospital setting. Aggressive rehydration lowers serum glucose independently of insulin. Clearly, additional glucose is not indicated for a patient with documented hyperglycemia. Furosemide (La ix), a loop diuretic, is contraindicated in patients with dehydration or hypovolemia.

A man has a severe headache and vomiting that has progressively worsened over the past 36 hours. Which of the following questions would be MOST important to ask him initia lly? A) Do you have a history of hypertension? B) Have you experienced a recent head injury? C) Do you have any abdominal pain or diarrhea? D) ls there a history of meningitis in your family?

B) Have you experienced a recent head injury? Feedback: A severe headache and vomiting that progressively worsens could indicate a subdural hemorrhage; therefore, one of the most important questions to ask the patient is if he experienced any recent head injury, even as far back as a week. Subdural hematomas are the result of venous bleeding and can be insidious in their presentation, with symptoms that often appear hours to days after the initial injury. When obtaining your SAMPLE history, you should specifically inquire about a history of hyperten ion. Meningitis is not a hereditary disease.

Which of the following is consistent with hyperglycemic ketoacidosis? A) Acute onset B) Hyperpnea C) Diaphoresis D) Bradypnea

B) Hyperpnea Feedback: When glucose does not reach the cell where it can be used for energy, the cell will metabolize fat instead, which produces ketoacids. As the body attempts to rid itself of these acids, the respirations become deep (hyperpnea) and rapid (tachypnea) with a noted fruity or acetone breath odor (Kussmaul respirations). Hyperglycemic ketoacidosis manifests over several hours to even a few days. As blood glucose levels rise, the kidneys attempt to excrete the excess glucose, taking water with it. This results in dehydration, which typically manifests with warm, dry skin that has poor turgor.

Which of the following analgesic medications would be appropriate when there is a concern for inducing respiratory depression or hypotension? A) fentanyl citrate B) Ketorolac tromethamine C) Meperidine hydrochloride D) Nalbuphine hydrochloride

B) Ketorolac tromethamine Feedback: Ketorolac tromethamine (Toradol} is a nonsteroidal anti-inflammatory drug (NSJ\JD). It is not a narcotic (opioid); therefore, it does not tend to cause hypotension or respiratory depression. However, it should be used with caution in patients with renal disease or patients who are bleeding (NSAIDs can increase bleeding time). All the other medications listed are narcotics and can cause hypotension and/or respiratory depression. Fentanyl citrate (Sublimaze) is a commonly used opioid agonise. I t is very potent, rapidly acting, and has a short half-life. Meperidine hydrochloride (Demerol) is a synthetic narcotic that is often given with hydroxyzine (Viscaril) to decrease nausea. Nalbuphine hydrochloride (Nubain) is al o a synthetic narcotic.

Which of the following describes the typical sequence of events that precedes cardiac arrest in a drowning episode? A) Dysrhythmias, laryngospasm, hypoxia B) Laryngospasm, hypoxia, dysrhythmias C) Laryngospasm, dysrhythmias, hypoxia D) Hypoxia, laryngospasm, dysrhythmias

B) Laryngospasm, hypoxia, dysrhythmias Feedback: Typically, when a swimmer becomes panicked, he or she starts swallowing large amounts of water. Even a small volume of aspirated water (fresh or salt) can cause laryngospasm, which leads to hypoxia and unconsciousness. As the hypoxia worsens, cardiac dysrhythmias can develop, which leads to cardiac arrest.

Overdose of which of the following drugs would likely respond favorably to naloxone? A) Adderall B) Meperidine C) Midazolam D) Phenobarbital

B) Meperidine Feedback: Meperidine hydrochloride (Demerol) is a potent narcotic analgesic; its central nervous system depressant effe cts can be reversed with the administration of naloxone. Naloxone (Narcan) is a narcotic antagonist that binds to opiate/opioid receptor s ites in die body, thus blocking the central nervous system depressant effects (ie, respiratmy depression, hypotension, bradycard ia) diat narcotics cause. It is important to note that the effects of many narcotic/opiate drugs outlive the effects of a single dose of naloxone; therefore, repeat doses of naloxone are often needed. Midazolam (Versed) is a benzodiazepine sedative-hypnotic and anticonvulsant; it is not a narcotic. Phenobarbital (Luminal) i a barbiturate sedative-hypnotic and anticonvulsant; it is not a narcotic. Adderall (amphetamine/dextroamphetamine) is a central nervous system stimulant used to treat ADHD; i t is not a narcotic.

A 70-year-old male presents with an altered mental status and severe weakness. According to his wife, he has had lower back pain and a fever for the past few days. His BP is 82/44 mm Hg, pulse is 160 beats/min, and respirations are 22 breaths/min. Which of the following treatments is indicated for him? A) Adenosine or a beta-adrenergic antagonist B) Normal saline boluses and norepinephline C) Sodium bicarbonate and calcium chloride D) Synchronized cardioversion with 50 joules

B) Normal saline boluses and norepinephline Feedback: The patient's vital signs clearly indicate shock, and given his recent illness and current presentation, the paramedic should suspect that he is septic. Septic patients need volume; in many cases, several liters of nuid are needed. A vasopressor (ie, norepinephrine [Levophed]) is often used in conjunction with n uid boluses, especially if hypotension persists despite fluid boluses. The patient's tachycardia is a compensatory response, not a primary dysrhythmia; therefore, action aimed at reducing his heart rate (ie, adenosine, beta blockers, cardioversion) are contraindicated. There is no evidence to indicate the need for sodium bicarbonate or calcium chloride.

Which of the following medications is classified as a tricyclic antidepressant? A) Fluoxetine hydrochloride B) Nortriptyline hydrochloride C) Buspirone hydrochloride D) Midazolam hydrochloride

B) Nortriptyline hydrochloride Feedback: Nortriptyline (Pamelor), amitriptyline (Elavil), and clomipramine hydrochloride (Anafranil) are commonly prescribed tricyclic antidepressant (TCA) medications. Fluoxetine hydrochloride (Prozac) is a selective serotonin reuptake inhibitor (SSRI) that is also used to treat depression as well as obsessive-compulsive disorder. Midazolam hydrochloride (Versed) is a benzodiazepine sedativehypnotic. Buspirone hydrochloride (Buspar) is an anxiolytic medication.

Which two cranial nerves are responsible for eye movement? A) Optic and oculomotor B) Oculomotor and trochlear C) Trigeminal and accessory D) Olfactory and hypoglossal

B) Oculomotor and trochlear Feedback: The oculomotor nerve (C Ill) and trochlear nerve (CN IV) both function to control movement of the eye, whereas the optic nerve (CN 11) controls vision. The trigeminal nerve (CN V) controls chewing, pain, temperature, and touch of the mouth and face. The olfactory nerve (CN I) controls one's sense of smell. The accessory nerve (CN XJ) controls movement of the head and shoulders. The hypoglossal nerve (CN XII) controls movement of the tongue.

The act of marking a foreign microbe or antigen for phagocytosis is called which of the following? A) Apoptosis B) Opsonization C) Degranulation D) Adaptive response

B) Opsonization Feedback: Opsonization is the act of marking a foreign microbe or antigen for phagocytosis or a dead cell for recycling. Surfactant in the lung can cover invading microbes and mark them for p hagocytosis as part of the innate immune response. Complement proteins can also cover or mark an invading microbe or antigen as part of the innate immune response. Antibodies can identify and mark an invading microbe as part of the adaptive immune response. /\poptosis is programmed cell death. Cytokines signal an infected or damaged host cell to die, preventing further escalation of infection to additional host cells. Degranulation is a cellular process that releases antimicrobial cytotoxic or other molecules from secretory ve icles called granules found inside some cells. It is used by several different cells involve in the immune response, including granulocytes (neutrophils, basophils, eosinophils) and mast cells. It is also used by certain lymphocytes such as natural killer ( K) cells and cytotoxic cells, whose main purpose is to destroy invading microorganisms. The adaptive immune response is the body's secondary response to infection. It has memory. It uses T cells (helper and killer) and B cells and their antibodies and the classic pathway of the complement cascade to accelerate of or potentiate the response to infection.

Which of the following signs are consistent with severe anemia? A) Flushed skin and bradycardia B) Pallor and shortness of breath C) Hematemesis and nosebleed D) Jaundice and abdominal pain

B) Pallor and shortness of breath Feedback: Anemia is a condition in which the blood lacks enough healLhy red blood cells (RBCs) or hemoglobin (Hgb). RBCs carry Hgb, and Hgb binds with oxygen. I[ RBC and Hgb counts are low, the oxyge ncarrying ability of the blood is reduced, and the cells of the body will not geL enough oxygen. Signs and symptoms of anemia include fatigue, tachycard ia, shortness or breath, pallor, and dizziness. Jaundice, a yellow t int to the skin, is associaLed with liver dysfunction. Hematemesis and epistaxis (nosebleed) can be caused by blood-clotting disorders (ie, thrombocyLDpenia, hemophilia).

A man presents with an acute onset of left -sided weakness, right-s ided facial droop, and slurred speech. What should you suspect? A) Right-side hemorrhagic stroke B) Right-side ischemic stroke C) Left-side ischemic stroke D) Left-side hemorrhagic stroke

B) Right-side ischemic stroke Feedback: Recalling that the right side or the brain controls the left side of the body and vice versa, this patient's clinical presentation is consistent with an ischemic stroke to the right cerebral hemisphere. lschemic strokes, caused by a blocked cerebral artery, typically present with unilateral weakness (hemiparesis) or paralysis (hemiplegia) on the opposite (contralateral) side or the stroke, a Facial droop on the same (ipsilateral) side as the stroke, and dysarthria (slurred or poorly articulated speech). Pupillary abnormalities, iF observed, typically occur on the ipsilateral side because of optic nerve crossover in the brain. The patient's clinical presentation is less consistent with a hemorrhagic stroke, which typically present with a sudden, severe headache; a rapid loss of con ciousness; and signs of increased intracranial pressure (ie, hypertension, bradycard ia, breathing abnormalities). A hemorrhagic stroke is the result of a ruptured cerebral artery (ie, an aneurysm).

At 2:30 AM, you respond to a crowded homeless shelter for a 52-year-old male who is sick. The patient complains of intense itching to his hands and axillae. Assessment of these areas reveals the presence of a rash. The patient denies any medical problems, but states that he was stung by a hornet 2 days ago. What should you suspect? A) Lice B) Scabies C) Herpes simplex D) Allergic reaction

B) Scabies Feedback: This patient's presentation is classic for scabies. Scabies are caused by infection with Sarcoptes scabiei, a parasite. Infection w ith scabies commonly affects families, children, sexual partners, and persons in communal living (ie, homeless shelters). Signs and symptoms of scabies include nocturnal itching and the presence of a rash involving the hands, nexor aspects of the wrists, axillary folds, ankles, toes, and genital area. Lice also present with i tching and irritation; however, unlike the nocturnal presentation of scabies, the symptoms of lice occur at any time of the day or night. Herpes simplex is characterized by small vesicles; a rash i s not common. It is highly unlikely that the patient is expe riencing an allergic reaction; he was stung by a hornet 2 days ago. An allergic reaction would have presented shortly following exposure, not 2 days later.

Increased parasympathetic tone, bradycardia, shunting of blood to the brain, and hypotension describes which of the following? A) Cushing reflex B) The diving reflex C) Beck reflex D) Cullen sign

B) The diving reflex Feedback: The diving reflex, also known as the mammalian diving reflex, is a protective mechanism of the body that is most prominent in cold temperatures (ie, falling in cold water). Through increased parasympathetic tone, the pulse rate and blood pressure both fall to decrease overall oxygen demand and consumption, while at the same time, blood is shunted to the brain to sustain it for as long as possible. The diving reflex is the reason why small children are able to survive for extended periods of time when submerged in cold water. The effect of the diving reflex dimi nishes with age. Cushing reflex, also called Cushing triad, is a trio of clinical signs in patients with increased intracranial pressure; it includes hyperten ion, bradycardia, and abnormal breathing. Cullen sign, bruising arou nd the umbilicus, is an indicator of blood in the peritoneal space. Beck triad, a trio of clinical sign obse,ved in patients with a severe pericardia! tamponade, includes muffled or distant heart tones, jugular venous distention, and a narrowing pulse pressure.

A 59-year-old male with a history of hypertension and diabetes presents with dark, tarry stools. He is confused; his blood pressure is 84/62 mm Hg and his pulse rate is 74 beats/min and weak. His medications include propranolol and Glucophage. Which of the following would MOST likely explain his heart rate? A) Increased parasympathetic tone in response to hypovolemia B) The prescribed adrenergic blocking agent to treat his hypertension C) Insulin suppressing his sympathetic nervous system D) Inadvertent overdose of his prescribed hypoglycemic medication

B) The prescribed adrenergic blocking agent to treat his hypertension Feedback: Adrenergic blocking medications, such as beta blockers, are commonly used to treat hypertension. Examples of beta blockers incl ude propranolol (lnderal), metoprolol (Lopressor), atenolol (Tenormin), and labetalol (Normodyne). Beta blockers suppress the sympathetic nervous system, which reduces heart rate and myocardial contractility. The patient in this scenario is in shock due to gastrointestinal bleeding. His heart rate, which you would expect to be fast, is in a normal range. This is most likely the result of a prescribed beta blocker medication, which is blunting the sympathetic nervous system's compensatory response to shock. I n hypovolemic shock, the sympathetic ne rvous system- not the para ympathetic nervous system-is stimulated, resulting in tachycardia, pallor, and diaphoresis, among other signs.

A patie nt presents with a diffuse petechial rash. What should yo u suspect? A) Leukopenia B) Thrombocytopenia C) Polycythemia vera D) Hemolytic anemia

B) Thrombocytopenia Feedback: Of the conditions listed, only one would cause a petechial rash. Thrombocytopenia, a reduction in the number of circulating platelets, can cause cutaneous bleeding and bleeding from the mucous membranes (ie, nosebleeds, rectal bleeding). Petechiae, tiny purple or red spots that appear on the skin, is caused by bleeding within the skin or under the mucous membranes. Localized petechiae may be harmless; however, a diffuse petechial rash indicates significant thrombocytopenia. Leukopenia is a reduction in the number of white blood cells (leukocytes); this condition places the patient at increased risk for infection. Polycythemia vera, also called primary polycythemia, is a hematologic disorder in which the bone marrow makes too many red blood cells; it may also result in an overproduction of white blood cells and platelets. Hemolytic anemia is a fonn of anemia caused by hemolysis, the abnormal breakdown (lysis) of red blood cells.

A woman with a history of Graves disease presents with an altered mental status. Her skin is hot to the touch and her pulse rate is 160 beats/min. What should you suspect? A) Myxedema B) Thyrotoxic crisis C) Addisonian crisis D) Cushing syndrome

B) Thyrotoxic crisis Feedback: Thyrotoxic crisis (thyroid storm, thyrotoxicosis) is a hypermetabolic clinical syndrome caused by critically high levels of the thyroid hormones T3 (triiodothyronine) and T4 (thyroxine). Signs and symptoms include high fever (as high as I 05'F to l06'F (40.5°C to 41.1°CJ), hypertension, and profound tachycardia. Untreated, it can lead to cardiac arrest. Thyrotoxic crisis may occur in conjunction with Graves disease, the most severe and common form of hyperthyroidism, or it may occur if a patient takes too much of their prescribed thyroid supplement (ie, levothyroxine). Advanced hypothyroidism (myxedema) is a hypometabolic clinical syndrome caused by a deficiency of T3 and T4; the patient's signs and symptoms are not consi tent with myxedema. Addisonian crisis, an acute manifestation of adrenal insufficiency, typically occurs after the abrupt cessation of corticosteroid therapy (ie, prednisone, hydrocortisone); th is would not explain the patient's hypermetabolic state. Cushing syndrome, caused by excessive cortisol production by the adrenal glands, can also cause a hypermetabolic state. However, given the patient's history of Graves disease, this is less likely.

In contrast to an anaphylactic reaction, an anaphylactoid reaction: A) is an immune response mediated by IgE antibodies. B) can occur without prior exposure to an offending agent. C) is usually less severe and does not require drug therapy. D) usually does not respond to antihistamine medications.

B) can occur without prior exposure to an offending agent. Feedback: Anaphylaxis is an immune response that is mediated by lgE antibodies, whereas an anaphylactoid reaction is an immune response that does not involve IgE antibody mediation. Anaphylaxis requires the patient to be sensitized first; he or she must be exposed to the offending agent (antigen) in order to bu ild up lgE antibodies. Because an anaphylactoid reaction is not lgE antibody mediated, prior exposure (sensitization) to the offending agent is not required. Examples of common agents that cause an anaphylactoid reaction include some contrast dyes given before radiography, morphine-derivative medications, and aspiri n . Even though the process that causes an anaphylactoid reaction is different from an anaphylactic reaction, the clin ical presentation, negative consequences (respiratory and circulatory failure), and treatment are the same. Do not discount a severe allergic reaction just because a patient, who presents with suggestive signs and symptoms, tells you that he or she was never exposed to the suspected trigger.

A patie nt with a blood glucose level of 650 mg/dL would be expected to present with: A) hypopnea, oliguria, abdominal pain, and vomiting. B) hyperpnea, dehydration, warm skin, and tachycardia. C) hypercarbia, anorexia, hyperactivity, and diaphoresis. D) tachypnea, anuria, alkalosis, and a bounding pulse.

B) hyperpnea, dehydration, warm skin, and tachycardia. Feedback: Patients with diabetic ketoacidosis (DKA) have a significantly elevated blood glucose level. In the absence of insulin, glucose cannot enter the cell; this results in metabolic acidosis secondary to ketoacid production from cellular fat metabolism. Excess blood glucose levels promote an osmotic diuresis, resulting in excessive urination (polyuria) and significant dehydration. Signs of dehydrat ion include warm, dry skin with poor turgor and a rapid, weak pulse. The respiratory buffer system attempts to eliminate ketones from the blood by increasing the rate and depth of breathing (tachypnea and hyperpnea); this is called Kussmaul breathing and would be expected to drive the patient's carbon dioxide level down (hypocarbia). A fruity or acetone breath odor classically accompanies Kussmaul breathi ng. Patients with DKA typically have a decreasing level of consciousness that progresses to coma; hyperactivity would not be an expected fi nding.

When released into the bloodstream, glucagon: A) directly increases circulating blood glucose levels. B) stimulates the liver to convert glycogen to glucose. C) stim ulates the liver to take in glucose and store it as glycogen. D) facilitates the cellular uptake of glucose for energy production.

B) stimulates the liver to convert glycogen to glucose. Feedback: When the body's blood glucose level falls, such as between meals, glucagon is released into the bloodstream in order to raise the glucose level. Glucagon, a hormone secreted by alpha cells of the islets of Langerhans (the endocrine component of the pancreas), stimulates the liver to convert glycogen to glucose - a process called glycogenolysis - and then releases it into the bloodstream. Through the process of glycogenolysis, glucagon indirectly increases circulating blood glucose levels. Fifty-percent dextrose (D50) is injected directly into the bloodstream; therefore, it directly increases circulating blood glucose levels. Insulin is respon ible for the removal of glucose from the blood for storage of glycogen, fats, and protein. When blood glucose levels are elevated, the beta cells of the islets of Langerhans secrete insulin, wh ich is carried by the bloodstream to the cells. The cells then take in more glucose a nd use it to produce energy. Insulin also stimulates the liver to take in more glucose and store it as glycogen for later use. Insulin is the only hormone that directly lowers blood glucose levels, and is essential for glucose to enter and nourish the cells. Once blood gl ucose levels have returned to normal, the islets of Langerhans decrease t11e secretion of insulin.

Forty-five minutes afte r being stung by a hornet, a woman presents with diffuse urticaria and itching. Her BP is 124/72 mm Hg, pulse is 88 beats/min and strong, respirations are 16 breaths/min and un labored, and oxygen saturation is 94%. What should you do? A) Administer 0.3 mg epinephrine I M B ) Administer oxygen via nasal cannula C) Administer 25 mg diphenhydramine IM D) Start an IV and give 500 mL of normal saline

C) Administer 25 mg diphenhydramine IM Feedback: The patient is clearly experiencing an allergic reaction; however, there is no evidence of hemodynamic or respiratory compromise ( in other words, anaphylactic shock). Given the information presented, the most appropriate course of action would be to administer 25 to 50 mg of diphenhydramine (Benadryl) via the intramuscular (IM) route. It certainly would not be unreasonable lo establish IV access and give the diphenhydramine by that route. In add ition, if her condition were to worsen, you would already have vascular access. Because there is no evidence of hemodynamic or respi ratory compromise, epinephrine is not indicated, nor are IV fluid boluses. Oxygen is not indicated (that's right, not even by nasal cannula) because her breathing is normal and her oxygen saturation is in a therapeutic range; oxygen will provide no benefit. It is important to note, however, that while most cases of anaphylaxis occur within m inutes, it can take up to an hour or more to develop. Therefore, you should closely monitor this patient during transport to the hospital.

A woman p resents with a severe migraine headache and nausea. Her vital signs are stable. What should you do? A) Defer analgesics until hospital arrival B) Apply chemical heat packs to her forehead C) Ask her if she has taken anything for her headache D) Administer 50 to 100 mcg of fentanyl IM or IV

C) Ask her if she has taken anything for her headache Feedback: Headache is a common patient complaint; most of the time, it is from a benign cause. However, you must still be cautious when caring for a patient with a headache because it cou ld indicate a more serious problem. A migraine headache is a complex condition thought to be caused by minor instability within certain clusters of neurons and also changes in the size of the blood vessels at the base of the brain. A migraine headache can last for several days. Begin your focused assessment by determining what, if anything, the patient has already taken and when they took it. Patients with migraine headaches are often prescribed specific medications to treat their condition, such a sumatriptan succinate (lmi trex) and sumatriptan plus naproxen (Treximet), among others. Analgesia, if needed, should not be with held U ust be sure to perform a careful assessment first). Medication options include ketorolac tromethamine (Toradol) 30 mg I M, meperidine (Demerol) 25 mg slow !VP, and fentanyl (Sublimaze) 50 mcg IV or IM. Also consider antiemetics, such as promethazine (Phenergan) 12.5 to 25 mg !VP or ondansetron (Zofran) 4 mg IV, IM, or by oral disintegrating tablet (ODT). Cool washcloths to the forehead or back of the neck may afford some relief. Many people with headaches also experience photophobia (l ight sensitivity); dimming the lights in the back of the ambulance can be helpful. Allow the patient to assume a comfortable position a nd transport to the hospital.

A man had a syncopal episode after several hours of vigorous exercise in the heat. He is responsive to pain only, and his skin is flushed, hot, and moist. What should you do? A) Establish IV access at once B) Take his temperature orally C) Begin rapid cooling measures D) Assess his blood glucose level

C) Begin rapid cooling measures Feedback: This patient is experiencing exertional heatstroke secondary to prolonged exposure to a hot environment. His markedly decreased level of consciousness and hot skin indicate a dangerously high core body temperature (CBT). Because the extent of brain damage depends on the severity and duration of hyperthermia, rapid cooling measures must begin immediately after you move him to a cooler environment. Othenvise, the patient's CBT will continue to rise and he will die. Remove the patient's clothing (it can trap heat), and begin fanning him while keeping his skin wet. Ice packs can be placed in his groin and axillae; however, you must not cool the patient to the point of shivering, as shivering produces heat and could increa e his CBT. Load the patient i nto the ambulance and begin rapid transport, conti nu ing cooling measures en route. Heatstroke is generally associated with volume depletion ; however, IV therapy should be performed en route to the hospital. You should assess the blood glucose level of any patient with an altered mental status, although this should not supersede immediate cooling measures. When assessing die patient's CBT, a hyperd1ermia thermometer (ie, a rectal probe) should be used, if available. Taking his temperature orally will yield little additional information. Unlike classic heatstroke, which commonly affects young chil dren, elderly patients, and those with significant medical problems, exertional heat troke does not always present with dry skin. Although the patient is not actively sweating, the skin is often moist from residual perspiration.

A 60-year-old woman, who has been taking high doses of prednisone for several months to treat her rheumatoid arthritis, presents with weakness and fatigue that has progressively worsened. On appearance, her face appears swollen. What should you suspect? A) Graves disease B) Addison disease C) Cushing syndrome D) Adrenal insufficiency

C) Cushing syndrome Feedback: Cushing syndrome is caused by excess cortisol production by the adrenal glands or by excessive use of cortisol or similar steroid (glucocorticoid) hormones. Tumors of the pitu itary gland or adrenal cortex can stimulate the production of cortisol, for example, and lead to Cushing syndrome. Administration of large amounts of corticosteroid hormones (ie, prednisone, methylprednisolone, dexamethasone, hydrocortisone) to treat condition such as rheumatoid arthritis, asthma, and systemic l upus can also cause Cushing syndrome. Regardless of the cause, excess cortisol causes characteristic changes in many body systems. Protein synthesis is impaired so that body proteins are broken down, which leads to loss of muscle fibers with resu ltant muscle weakness. Common signs and symptoms of Cushing disease incl ude weakness and fatigue; depression and mood swings; darkening of the skin (acanthosis) on the neck; and weight gain, especially on the abdomen, face ("moon face"), neck, and upper back. Primary adrenal insufficiency, also known as Addison disease, is caused by atrophy or destruction of the adrenal glands, leading to a deficiency of all the steroid hormones produced by these glands. Grave disease, a d isease process associated with hyperthyroidism, occurs when the thyroid gland produces excess thyroid hormones.

A man with type I diabetes has not taken his insulin in 3 days. Which of the following would you expect to e ncounter? A) Decreased urination; muscle tremors; diaphoresis B) Rapid, shallow breathing; headache ; hypoglycemia C) Dehydration; deep, rapid breathing; warm, dry skin D) Loss of appetite; pale, clammy skin; slurred speech

C) Dehydration; deep, rapid breathing; warm, dry skin Feedback: Insulin lowers serum glucose levels; therefore, if a patient did not take their insulin, you would expect the blood glucose level to be high (hyperglycemia). Hyperglycemia causes an osmotic diuresis, which leads to significant dehydration. Signs and symptoms of dehydration include tachycardia; warm, dry skin; poor skin turgor; and dry mucous membranes. As glucose pools in the blood, less is allowed into the cells; as a result, fat metabolism ensues and the patient develops ketoacidosis. As a compensatory mechanism, the respiratory system attempts to eliminate these ketoacids, so the patient's breathing becomes deep and rapid (Kus maul breathing). In contra t to ketoacidosis (caused by hyperglycemia), hypoglycemia, which can rapidly progress to insulin hock, occurs when a patient takes too much of their prescribed insulin, or cakes a regular dose of in su lin but does not eat. Signs and symptoms of hypoglycem ia include altered m ental status; rapid , shallow breathing; and cool, pale, moist skin.

A woman complains of severe neck pain. Her head is turned to the side and appears locked in that position. You also note that her eyes are clenched shut. She is conscious, but agitated. She started taking lithium the day before after being diagnosed with bipolar disorder. Which of the following medications is indicated? A) Naloxone B) Haloperidol C) Diphenhydramine D) Calcium chloride

C) Diphenhydramine Feedback: The patient is experiencing drug-induced dystonia. Dystonia is marked by severe muscle spasms that cause bizarre contortions, repetitive motions, or postures (localized or diffuse). Patients who take amipsychotic or neuroleptic drugs may experience an acute onsel of bizarre contortions of the face or body (secondary dystonia), usually wilhin 12 to 36 hours after a new drug is started or the dose of an existing drug is increased. Drugs commonly associated with dystonia include lithium, phenyloin (Dilantin), antidepre sanes, reserpine (Serpalan), and carbamazepine (Tegretol), among others. Nume rous patterns of drug-induced dystonia exist; lhi patient has lWO of the most common: blepharospasm (eyelid muscle spasm thal causes forced eyelid closure) and lorticollis (neck muscl e spasms that force the head to the side). Drug-induced dystonic reactions are treated with diphenhydramine (Benadryl), 50 mg IV over 2 minutes (repeat in 15 to 30 mi nutes if needed). Dopamine blockade by anti psychotics disrupts the ratio of acetylcholine and dopamine in the brain; anticholinergic drugs (eg, diphenhydramine) may restore this ratio. Haloperidol (Haldol), an antipsychotic drug, may cause dystonia (or in this case, exacerbate it) and is clearly contraindicated. aloxone (Narcan) is an opioid receptor antagonist used to reverse respiratory depression caused by opioid (narcotic) overdose, and calcium chloride is used to lreal hyperkalemia and calcium channel blocker overdose; neither is used to treat dystonia.

Signs and symptoms of insecticide poisoning include which of the following? A) Extreme hyperactivity and pupillary dilation B) Acute urinary retention and abdominal pain C) Excessive salivation and severe bradycardia D) A dry cough, tachycardia, and hypertension

C) Excessive salivation and severe bradycardia Feedback: Organophosphates, such as what is found in pesticides/insecticides and chemical nerve agents (VX, sarin, tabun, soman), deactivate acetylcholinesterase (AChE), an enzyme that regulates the degradation of acetylcholine (ACh). ACh is the chemical neurotransmitter of the parasympathetic nervous system. Without AChE, there is nothing to regulate ACh degradation; this would cause cardiovascular collapse secondary to massive parasympathetic stimulation. The clinical presentation of severe organophosphate toxicity can be recalled using the mnemonic DUMBE LS, which stands for defecation, urination, miosi (pupillary constriction), bronchorrhea and bradycardia, eme is, lacrimation, and alivation. Treatment includes atropine sulfate, which blocks the effects of ACh , and pralidoxime chloride (2-PAM , Protopam), which reactivates ACh E. A commercial auto - injector (DuoDote) is ava ilable; it contains both pralidoxime and atropine.

A woman was stung by a scorpion. Within a few minutes, she developed nausea and abdominal pain. She is confused, her skin is pale, and her BP is 80/50 mm Hg. What s hould you do? A) Give 4 mg ondansetron I M B ) Give 25 mg d iphenhydramine IM C) Give 0.3 to 0.5 mg epinephrine IM D) Start an IV and give 125 mg methylprednisolone

C) Give 0.3 to 0.5 mg epinephrine IM Feedback: This is anaphylaxis' There is a known exposure and the patient is in shock. Do not let the absence of urticaria (hives) or angioedema fool you; these are not always present. Nausea and abdominal pain are actually quite common. The MOST critical therapy for anaphylaxis is epinephrine, and there are O contraindications to its use in anaphyl axis. Of the options listed, IM epinephrine is the best choice. IM epinephrine should be used as a bridge to IV epinephrine. Get epinephrine on board by the most rapid means, and then establish vascular access. If die patient's condition has not resolved or is getting worse, an epinephrine infusion may be needed. Through its alpha-I and beta-2 a d renergic effects of vasoconstriction and bronchodilation, epinephrine reverses the hypotension and bronchospasm associated with anaphylactic shock. Diphenhydramine (Benadryl) is given after epi nephrine to block the release of histamines that are causing the allergic reaction; the adult dose is 25 to 50 mg IM or IV. Methylpredni alone (Solu-Medrol), a glucocorticoid (steroid) anti-innammatory drug, may be given as an adjunct to epinephrine and diphenhydramine; the adult dose is 125 mg.

A 71-year-old woman with renal disease presents with confusion, severe weakness, shortness of breath, and peripheral edema. She is unable to tell you when she had her last dialysis treatment. Her BP is 140/84 mm Hg, pulse is 55 beaths/min and weak, and respirations are 22 breaths/min. Her 12-lead ECG reveals a fi rst-degree AV block with wide QRS complexes. What should you do? A) Give 0.5 mg atropine B) Give 40 mg furosemide C) Give 1 g calcium chloride D) Give 150 mg amiodarone

C) Give 1 g calcium chloride Feedback : You should assume that the patient has missed some of her d ialysis treatments. These patients of Len present with signs of volume overload (ie, shonness of breath, peripheral edema). If toxins have accumulated, the patient may have u remic frost, confusion, and muscle twiLching. Most patients with end-stage renal disease (ESRD) will have some form of coagulopaLhy, and easy bruising may be apparent. Weakness and faLigue can be chronic and pronounced, and mosl patients with ESRD will be anemic. Eva luate the ECG for signs of hyperkalemia, particularly if dialysis was missed. Hyperkalemia can cause virtually any dysrhythmia, although the most common signs are tall, peaked T waves (early); PR prolongation, and QRS widening. Eventually, the QRS complexes become so wide that a sine wave pattern develops. Hyperkalemia is a preventable cause of dealh, and the paramedic should not wait until pronounced ECG signs are present. Of the treatments listed, calcium chloride is the most critical drug to give her and can literally save her life; give I g ini tially and repeat as necessary. Sod i u m bicarbonate should also be con idered, but not before calcium. Diuretic therapy may be initiated in the prehospital setting, bul it is more commonly given at the hospital. The patient has a relatively slow heart rate; however, it is not related to vagal induction, but rather another manifestation of hyperkalemia (hyperkalemic patients can be normocardic, bradycardic, or tachycardic). Atropine would be of no real benefit to her. Do OT give the patient a sodium channel blocking drug, such as amiodarone or lidocaine. Hyperkalemia causes sodium channel blocki ng effects; sodium channel blocking drugs could cause cardiac arrest.

A 26-year-old female who overdosed on amitriptyline is responsive to pain only. Her BP is 70/40 mm Hg and her heart rate is 140 beats/min. A 12- lead ECG reveals wide QRS complexes and a prolonged QT interval. What should you do? A) Perform cardioversion at 100 joules B) Administer 0.2 mg flumazenil IV pu h C) Give 20-mL/kg crystalloid fluid boluses D) Administer 150 mg amiodarone over 10 minutes

C) Give 20-mL/kg crystalloid fluid boluses Feedback : Tricyclic antidepressants (TCAs), such as amitriptyline (Elavil) and nortriptyline (Pamelor), affect the body through several mechanisms. Patients with toxic ingestions are likely to present with tachycardia, tachypnea, hypotension, and hyperd1ermia. Altered mental status, delirium, and seizures can occur following a toxic ingestion. In addition, because TCAs possess sodium and potassium channel blocking properties, a variety of ECG changes can occur; some of them can be lethal. ECG changes include PR prolongation , QRS widening, and QT prolongation. In this scenario, the patient is significantly hypotensive; this should be treated with sequentia l fluid boluses of 20 mL/kg (many patients are refractory to a singl e bolus). Use caution with fluids, however, because pulmonary edema can occur in TCA overdose. Although not commonly carried in the prehospital setting, flumazenil (Romazicon), a benzodiazepine antagonist, is contraindicated in patients with TCA overdose. TCAs are epileptogenic (they cause seizures); if concomitant overdose with a benzodiazepine has occurred, flumazenil can unmask seizures. If QRS widening is noted, a i s the ca e with this patient, consult with medical control or follow you r local protocols regardi ng the administration of sodi u m bicarbonate. Do NOT give amiodarone or lidocaine to patients with TCA overdose; both of these drugs possess sodium channel blocking properties and can cause cardiac arrest. Cardioversion would, in all likelihood, be an ineffective therapy because the patient's tachycardia is secondary to TCA toxicity; it is not a primary dysrhythmia.

A 74-year-old woman with a recent urinary tract infection has an altered mental status. Her BP is 84/50 mm Hg, pulse is 84 beats/min, respirations are 24 breaths/min, and blood glucose is 82 mg/dL. What should you do? A) Begin an antib iotic infusion B) Begin a norepinephrine infusion C) Give a 30-mL/kg crystalloid bolus D) Give 12.5 grams dextrose JV push

C) Give a 30-mL/kg crystalloid bolus Feedback : When an older person presents with an altered mental status, sepsis should be high on your differential diagnosis. This is especially true when there is a source for sepsis, such as an infection of the respiratory system or urinary tract. Several criteria exist that can be used to identify a septic patient. The Sequential (Sepsis-related) Organ Failure Assessment (SOFA) score is one such system and is commonly u sed in the intensive care unit (ICU). The SOFA score was modified for use outsi de the ICU, such as the fast-paced emergency environment in both the prehospital setting and initial presentation in the emergency department. In these environments, the modi fied qSOFA (or quickSOFA) is u ed. The qSOFA criteria i nclude a respiratory rate greater than 22 breaths/min, systolic BP less than 90 mm Hg, and altered mental status. The patient in this scenario meets all three of the qSOFA criteria. Hypotensive septic patients need volume! Recommended resuscitation is rapid infusion of 30 mL/kg of crystalloid fluids until any one of the following occur: mean arterial pressure (MAP) exceeds 65 mm Hg; presentation or worsening of crackles in the lungs; a decrease in oxygenation, espec ially if accompanied by an increase in difficulty in ventilation; or a serum lactate of less than 2 mmoJ/L is achieved (not typically measured in the prehospital setting). Vasopressor medications are i ndicated for patients who remain hypotensive after initial fluid resuscitation. orepinephrine is the vasopressor of choice for patients in septic shock; like fluid resuscitation, the goal of vasopressor therapy is to achieve a MAP of at least 65 mm Hg. IV dextrose is not indicated for this patient because she is normoglycemic.

A 68-year-old man returned to a skilled nursing facility 3 days ago after having his Foley catheter changed. Today, he is found to be confused. His BP is 70/40 mm Hg, pulse is 110 beats/min and weak, respirations are 28 breaths/min, and his skin is cold to the touch. What should you do? A) Administer an antibiotic B) Begin a norepinephrine infusion C) Give a 30-mL/kg crystalloid bolus D) Start a dopamine infusion at 2 mcg/kg/min

C) Give a 30-mL/kg crystalloid bolus Feedback: This patient has a probable source of infection, his Foley catheter. Not only is he septic, but he is in septic shock' He would receive a score of 3 on the qSOFJ\ score (GCS less than 15, respiratory rate greater than or equal to 22 breaths/min, systolic BP less than or equal to 110 mm Hg). If sepsis-induced hypoperfusion is present, the Surviving Sepsis Campaign recommends the administration of at least 30 ml/kg of IV cry talloid within the first 3 hours. Many patients with sep is do not receive adequate volume. In addition to monitoring the patient's oxygen saturation, ETCO2, BP, and heart rate, carefully monitor him for increased work of breathing or the development of crackles. If hypotension persists despite fluid administration, a vasopressor should be initiated. Norepinephrine (Levophed) has been found to be effective in treating fluid-refractory septic shock. Dopamine at 2 rncg/kg/min will have no effect on the patient's cardiac output. Antibiotics such as cefepime (Maxipime) or ceftriaxone (Rocephin) may play a role in the prehospital management of sepsis, but do not take priority when the patient is in shock. Follow your local protocols.

A man complains of difficulty breathing and a fever. He is emaciated, is coughing, and has purple blotches on his trunk. What should you suspect? A) Tuberculosis B) Pneumonia C) HIV/AIDS D) Hepatitis

C) HIV/AIDS Feedback: Signs and symptoms of advanced HIV disease (AJDS) include weight loss, which gives the patient an emaciated appearance; persistent fever; night sweats; fatigue; and purple blotches on the skin, which are malignant lesions called Kaposi's sarcoma. The patient may have pneumonia; however, this would be the result of the immunosuppression associated with HIV/ AIDS. Kaposi's sarcoma are not observed in patients with tuberculosis, pneumonia (without HIV/AJDS), or hepatitis.

A 30 -year-old man collapsed at a local park. He is disoriented, his skin is hot and moist, and his respirations are rapid and shallow. What should you suspect? A) Heat exhaustion B) Heat prostration C) Heatstroke D) Heat cramps

C) Heatstroke Feedback: Hot (dry or moist) skin, tachypnea, and an altered mental status are indicative of heatstroke. Two major factors help di fferentiate heat exhaustion from heatstroke. In heat exhaustion, sometimes called heat prostration, the patient is usually conscious and alert with cool, moist sk in. In heatstroke, the patient's mental status is typically altered and their skin is hot to the touch. Although the patient in heatstroke is not actively sweating, they may have residual perspiration on the skin; this is especially true in patients with exertional heatstroke. Do NOT rule out heatstroke just because the patient's skin is moist.

A patient with diabetic ketoacidosis would be expected to present with which of the following clinical signs? A) Hypoglycemia and dehydration B) Hypoglycemia and polyuria C) Hyperglycemia and dehydration D) Hyperglycemia and oliguria

C) Hyperglycemia and dehydration Feedback: Diabetic ketoacidosis (DKA), also called hyperglycemic ketoacidosis, is characterized by hyperglycemia, polyuria (excessive urination), polydipsia (excessive thirst), and polyphagia (excessive hunger). Other findings include warm, dry skin; dehydration; and deep, rapid respirations ( Kussmaul respirations). The progression to DKA is typically slow, often over several hours to a few days. By contrast, insulin shock, caused by severe hypoglycemia, is characterized by a rapid onset, often within a few minutes.

Signs of myxedema may include which of the following? A) Hyperactivity B) Tachycardia C) Hypothermia D) Weight loss

C) Hypothermia Feedback: Advanced hypothyroidism is sometimes called myxedema. Frequently, patients have localized accumulations of mucinous material in the skin, which gives the disease its name (the prefix myxrefers to ·mucin," and edema means "swelling"). Myxedema manifests as a general slowing of the body's metabolic processes due to a significant reduction or absence of the thyroid hormones T3 (triiodothyronine) and T4 (thyroxine). Since the thyroid gland regulates the metabolic rate and metabolism produces heat energy, pat ients with myxedema are prone to hypothermia. This also explains why patients with hypothyroidism are poorly tolerant of cold temperatures. Other signs and symptoms of myxedema include lethargy, depre sion, bradycardia, and weight gain. I n severe cases, coma and death can occur.

Which of the following conditions wo uld produce the MOST rapid loss of consciousness? A) Ketoacidosis B) lschemic stroke C) Insulin shock D) Hyperglycemia

C) Insulin shock Feedback: Insulin shock (hypoglycemic crisis) is most noted for its rapid onset of symptoms, which includes loss of consciousness. Hyperglycemia, which can lead to ketoacidosis, typically presents over a period of hours to days. Unlike a hemorrhagic stroke, an ischemic stroke generally does cause a rapid or immediate loss of consciousness.

A 66 -year-old man presents with acute aphasia, facial droop, and left-side paralysis. He is ignoring the left side of his body as though it does not exist. His BP is 144/84 mm Hg, pulse is 76 beats/min, and respirations are 14 breaths/min. What should you suspect? A) Hemorrhagic stroke B) Transient ischemic attack C) Large vessel occlusive stroke D) Transient cerebral artery spasm

C) Large vessel occlusive stroke Feedback: The patient's symptoms are ominous and indicate a major stroke. A large vessel occlusion (LVO) is a type of ischemic stroke that results from a blockage in a major cerebral artery, such as the basilar artery, internal carotid artery terminus, or middle cerebral artery. When these major arteries are blocked, significant portions of the brain are deprived of oxygen, resulting in higher-order brain function disruptions. There are several screening cools to assess for LVO stroke in the preho pita! setting; however, one specific screening tool has not been found to be superior. On such screening tool is called the VA assessment (Vision, Apha ia, Neglect). If a patient does not have hemiparesis or hemiparalysis, thenthey are said to be VAN negative and the l ikelihood of an LVO stroke is low. However, if hemiparesis or hemiparalysis is present, the presence of visual disturbances, aphasia (expressive or receptive), or neglect increases the likelihood of an LVO stroke. The patient in this scenario is aphasic and is ignoring one side of his body, as though it did not exist (neglect). Although a hemorrhagic stroke cannot be ruled out in the field, the patient's symptoms are more consistent with ischemic stroke. Furthermore, hemorrhagic stroke commonly presents with signs of increased intracranial pressure (ie, hypertension, bradycard ia, vomiting). Transient ischemic attack or transient cerebral artery spasm is unlikely because there is no evidence that the patient's symptoms are resolving.

Which of the following statements regarding lightning injuries is correct? A) Asystole following a lightning strike is generally intractable and responds poorly to CPR. B) Muscle that is injured by lightning produces methemoglobin, which can damage the kidneys. C) Lightning injuries tend to resemble blast injuries more than they do high-voltage injuries. D) Lightning is alternati ng current that causes severe damage to the nerves and microvasculature.

C) Lightning injuries tend to resemble blast injuries more than they do high-voltage injuries. Feedback: Lighting carries enormous electrical current; i ts energy can reach JOO million volts, and peak currents can be in the range of 200,000 amps. Unlike other high- voltage electrical current, lightning is direct - not alternating - current, and the duration is measured in milliseconds. Thus, l ightning injuries tend to resemble blast injuries more than they do high-voltage injuries, with damage to the tympanic membranes and air-containing internal organs being common. Lightning delivers a massive direct-current shock that depolarizes the entire heart, usually resulting in asystole. However, the heart may resume beating pontaneou ly shortly after the shock or after a few minutes of high-quality CPR. It i for thi s reason that pulseles and apneic patients who have been struck by lightning shoul d be treated first (reverse triage); victims who are conscious following a lightning strike, although injured, are less likely to develop cardiac arrest. Muscle damage may occur following a lightning strike, and the release of myoglobin from injured muscle can damage the kidneys. Methemoglobin - an abnormal Forni of hemoglobin - occurs when hemoglobin is oxidized during blood decomposition or by the action of various toxic agents (ie, nitrates). Methemoglobin contains iron in the ferric state and is unable to carry oxygen to the body's tissues.

Which of the following parameters is included in the quick Sequential (Sepsis-related) Organ Failure Assessment (qSOFA) score for sepsis? A) Heart rate B) Temperature C) Mental status D) Leukocyte count

C) Mental status Feedback: During the Third International Consensus Definitions for Sepsis and Septic Shock (Sepsi -3, or Sep-3), it was recommended that "sepsis should be defined as life-threatening organ dysfunction caused by a dysregulated host response to infection." Sepsis-3 also refuted the previously held belief that there was progression on a continuum from sepsis to severe sepsis to septic shock, and the term "severe sepsis" was eliminated. The Systemic Inflammatory Response Syndrome (SIRS) criteria were previously used to identify patients with sepsis; they consisted of assessments of temperature, heart rate, re piratory rate, and white blood cell count. However, sensitivity and specificity of these criteria were low, so the Sep-3 indicators instead focused on prognostic indicators; they showed that if a patient had changes in certain organ systems, such as hepatic, renal, cardiovascular, and respi ratory function, they had an increased risk of death. Enter the Sequential (Sepsis- related) Organ Failure Assessment (SOFA) Score, which is used to identify patients with sepsis in the intensive care unit. This score has been further modified for use in the emergency department and prehospital setting. In these settings, the modified qSOFA (or quickSOFA) is used. qSOFA criteria include respirato,y rate greater than 22 breaths/min, systolic BP less than 90 mm Hg, and altered mental status. Regardless of the criteria or scoring system u sed, the earlier a sepsis patient can be identified and treated, the better the outcome.

Which of the following ECG abnormalities is associated with hypothermia? A) Sine wave B) Delta wave C) Osborn wave D) Flat T waves

C) Osborn wave Feedback: An Osborn wave (J wave) may be observed on the ECG of a patient with significant hypothermia. An Osborn wave is a positive denection that causes elevation of the J point (intersection of the QRS complex and ST segment) above the isoelectric line. A sine wave is characterized by a markedly widened QRS that merges with its corresponding T wave; sine waves are ECG indicators of severe hyperkalemia. Flattened or inverted T waves, prominent U waves, and a wide PR interval, are ECG indicators of hypokalemia. Della waves are characterized by a slurring of the initial part of the QRS complex; they are seen in patients with Wolff-Parkinson -White (WPW) syndrome, a ventricular preexcitation syndrome.

A 34-year-old woman presents with acute dyspnea and anxiety. Her skin is pale and diaphoretic, her heart rate is 130 beats/min, and her end-tidal CO2 is 14. You are unable to obtain a signal on the pulse oximeter. What should you suspect? A) Anxiety attack B) Viral bronchitis C) Pulmonary embolism D) Severe ketoacidosis

C) Pulmonary embolism Feedback: Of the diagnoses listed, the patient's presentation most closely resembles acute pulmonary embolism. Of course, it could be an anxiety attack; however, this should never be a primary diagnosis-it is a diagnosis of exclusion. Ketoacidosis does not present with acute symptoms; they usually develop over several hours to a few days. Furthermore, ketoacidosis is typically associated with marked dehydration secondary to hyperglycemia; you would expect a dehydrated patient's skin to be warm and dry, not pale and diaphoretic. Bronchitis classically presents with a productive cough, which there is no mention of, and it rarely presents acutely. Acute dyspnea (with or without chest pain), tachycardia, and a low end-tidal CO2 (from poor perfusion) should put acute pulmonary embolism at the top of your differential diagnosis.

What cranial nerve is assessed by asking the patient to shrug their shoulders and turn their head from side to side? A) Trigeminal B) Vestibulocochlear C) Spinal accessory D) Glossopharyngeal

C) Spinal accessory Feedback: If lime and patient condition permits, assessment of the 12 pairs of cranial nerves should be performed. The spinal accessory nerve (XI), a motor nerve, controls shoulder and neck movements. Asking the patient to shrug their shoulders and turn their head from side to side assesses cran ial nerve XI. The trigeminal nerve (V) provides motor control to the muscles of chewi n g and sensory control to the face, sinuses, and teeth. Asking the patient to clench their teeth and then lightly stroking your finger over their forehead and c heeks and asking them to identify where they are being touched assesses cranial nerve V. The vestibulocochlear nerve (VIII), a sensory nerve, controls hearing and balance perception. Checking a patient's hearing and asking them to stand on one leg (if safe to do so) assesses cranial nerve VI I I . The glossopharyngeal nerve (IX) provides motor control to the throat and swallowi ng mechanism and sensory control to the tongue, throat and ear. Asking the patient to swallow assesses cranial nerve IX. Re fer to your paramedic text regarding assessment of all of the cranial nerves.

A woman swallowed a lot of pills. Which of the following should you inquire about FI RST? A) A history of psychiatric care B) When the pills were taken C) What kind of pills were taken D) The patient's weight in kilograms

C) What kind of pills were taken Feedback : After ensuring scene safety, your initial a Lion when caring for any patient is to ensure a patent airway and adequate breathing. After doing this, your next action for this patient should be to determine what kind of pills she took, then determine how long ago she took the pills. This information will enable you to provide the most appropriate treatment, which may include an antidote or reversal agent. Determining the patient's weight in kilograms is also important as many antidotes are weight-based. When you have determined what was taken, when it was taken, and the patient's weight in kilograms, contact medical control or the poison control center. Whether or not the patient has a psychiatric history will not innuence the immediate care chat you provide, although you should inq u i re about this at some point.

Disequilibrium syndrome manifests with signs and symptoms of: A) severe hypokalemia. B) high serum potassium levels. C) increased intracranial pressure. D) excessive catecholamine release.

C) increased intracranial pressure. Feedback: Duri ng dialysis, the concentration of urea in the blood is lowered rapidly, while the solute concentration of the cerebrospinal nuid (CSF) remains high. Water, of course, moves by osmosis from a solution of lower concentration into a solution of higher concentration. Thus, as a consequence of dialysis, water initially shifts from the bloodstream into the CSF, thereby increasing intracranial pressure (ICP). In such a circumstance, the pat ient may experience typical signs and symptoms of mildly increased ICP, including nausea, vomiting, headache, and confusion. After a few hours, the n uid will re-equilibrate between the bloodstream and CSF, and the patient's symptom will spontaneou ly resolve. In the field, however, it may be impossible to d istinguish between disequilibrium syndrome and subdural hematoma, to which dialysis patients are particularly prone. Signs and symptoms of hyperkalemia (high serum potassium levels) a re commonly encountered in patients with chronic renal failure, especially if the patient's potassium intake is increased or if a dialysis treatment is missed. Hyperkalemic patients commonly present with profound muscular weakness; on the ECG, peaked T waves and prolonged QRS complexes are commonly observed. Hypokalemia (low serum potassium level) may occur as a consequence of overaggressive dialysi , and would most likely be encountered during or i mmediately following a dialysis cycle. Hypokalemic patients are o ften hypotensive and commonly present with cardiac dysrhychmias (usually bradycardias). Tachycardia and hypertension-signs of excessive catecholamine release-are not commonly associated with dialysis.

Polycythemia is a condition that results in: A) lactic acidosis due to anaerobic metabolism. B) a marked increase in the core body temperature. C) increased oxygen-carrying capacity of the blood. D) spontaneous bleeding due to a low platelet count.

C) increased oxygen-carrying capacity of the blood. Feedback: Polycythemia is denned as an increase in red blood cell production. It often occurs in response to hypoxia. but may occur for other reasons. Patients wid1 emphysema, for exa mple, are orten polycythemic in response to chronic hypoxia; this is why they are commonly re[erred to as "pink puffers.· Because red blood cells attach to the hemoglobin molecule and cany oxygen, polycythemia increases the oxygen-canying capacity and erficiency of the blood. Of course, how well the tissues actually get oxygenated depends on the oxyhemoglobin saturation. A marked increase in core body temperature results from environment factors (eg, heatstroke), inflammation, and infection. Pyrogens are fever-causing agents that are produced by the immune system. When an infectious or inflammatory process exists, the body produces excess levels of pyrogens, resulting in pyrexia (fever). Patients with thrombocytopenia - a reduction in platelets - commonly experience spontaneous bleeding. Furthermore, thrombocytopenic patients have impaired hemostasis; even minor i nternal bleeding can be extremely serious. Aerobic metabolism is the normal metabolic process; it produces carbon dioxide and water as its byproducts. In the absence of oxygen, the cells convert from aerobic to anaerobic metabolism and produce lactic acid.

Easy bruising, lymph node enlargement, and splenomegaly are c linical signs of: A) anemia. B) lymphoma. C) leukemia. D) polycythemia.

C) leukemia. Feedback: Leukemia is cancer of the blood, and is caused by an abnonnal proliferation (production by multiplication) of leukocytes (white blood cells) in the bone marrow. Leukemic c ells impair the normal production of red blood cells (RBCs), white blood cell s (WBCs), and platelets (thrombocytes); this results in anemia, leukopenia (low WBC count), and easy bleeding due to thrombocytopenia (low platelet count). In leukemia, excessive white blood cells accumulate in major organs (ie, spleen, liver, brain, and lymph), causing them to become enlarged (ie, splenomegaly [enlarged spleen], adenopath y [enlarged lymph nodes), hepatomegaly [enlarged liver)). Other igns and symptoms of leukemia include bone pain (due to increased pressure in the medullary canal of the bone), fever, fatigue, night sweats, and weight loss.

You are assessing a patient who presents with a melena and abdominal pain. Which of the following medical history findings is the MOST clinically significant? A) Hype rtension B) Hashimoto disease C) von Willebrand disease D) Type II diabetes mellitus

C) von Willebrand disease Feedback: All of the diseases listed are clinically significant. However, the presence of von Willebrand disease in a patient with internal or external bleed ing should concern you the most. Von Willebrand disease, a bleeding disorder similar to hemophilia, is caused by a deficiency of von Willebrand factor (vWF), a blood clotting protein. vWF circulates attached to factor VIII, another blood clotting protei n. As a consequence of von Willebrand disease, the normal process of hemostasis is interrupted and the patient will continue to bleed. Hashimoto disease, also called Hashimoto thyroiditis, is a cause of hyperthyroidism; i t i an autoimmune disorder that affects the thyroid-stimulating hormone (TSH) receptors. While hypertension, Hashimoto disease, and type II diabetes can certainly complicate the clinical picture of any sick or injured patient, bleeding disorders in a patient with active bleeding would clearly be the most detrimental.

An unresponsive woman , who is a known IV drug abuser, has slow, shallow breathing and a slow, weak pulse. After initiating bag-mask ventilation, what intervention should be performed next? A) 500-mL N fluid bolus B) 0.5 mg of atropine IV push C) Transcutaneous cardiac pac ing D) 0.4 to 2 mg naioxone IM or IN

D) 0.4 to 2 mg naioxone IM or IN Feedback: The patient has signs of central nervous system depression (unresponsive, hypoventilation, bradycardia), and given her history of IV drug abuse, you should suspect that she overdosed on heroin. The most immediate priorities are to ensure airway patency and begin ventilating with a bag-mask device (opioid overdose death is the result of hypoxia and acidosis). Naloxone (Narcan) should then be given in a dose that improves the patient's breathing; this is typically in a range of 0.4 to 2 mg. Because she is a known IV drug abuser, consider starting on the lower dosing end (0.4 to 0.8 mg). Naloxone can be given IV, I M , or 11 . Atropine, a parasympatholytic drug, would have minimal to no benefi t on the patient's heart rate because her bradycardia is not caused by increased vagal tone. Transcutaneous cardiac pacing (TCP) would also be of minimal benefit. IV fluid boluses are often not needed for patients who overdosed on an opioid. However, if her BP is low (after improving her breathing and heart rate), a fluid bolus should be considered.

You are setting up a norepinephrine infusion to run at 5 mcg/min. You add 8 mg of norepinephrine to 500 mL of normal saline. How many milliliters per hour will be delivered? A) 8 B) 11 C) 15 D) 19

D) 19 Feedback: The concentration yielded when you add 8 mg (8,000 mcg) of norepinephrine to 500 mL of normal saline is 16 mcg/mL. A desired dose of 5, divided by the concentrat ion on hand of 16, equals 0.3125 mL/min; multiplied by 60, this yields 18.75 (19) mL per hour.

What is the correct dose and route of diphenhydramine for a patient experiencing an allergic reaction? A) 0.3 to 0.5 mg IM B) 2 to 10 mcg/min IV C) 1 to 2 mg/kg IM or IV D) 25 to 50 mg IM or IV

D) 25 to 50 mg IM or IV Feedback: Diphenhydramine (Benadryl) is an HI receptor antagonist (antihistamine). It is used to treat allergic reactions (mild, moderate, and severe). The dose is 25 to 50 mg and can be given via the IM or IV route. In mild or moderate reactions, diphenhydramine may be the only medication needed. In severe reactions (ie, anaphylaxis), diphenhydramine is an adjunct medication to epinephrine. Remember, the MOST imponant treatment for anaphylaxis is epinephrine! The dose is 0.3 to 0.5 mg IM; in refractory cases, an epinephrine infusion at 2 to IO mcg/min may be needed.

A 30 -year-old man has been having a seizure for the past 15 minutes. What drug/dose combi nation should you give? A) 25 g dextrose B) 100 mg thiamine C) 0.4 to 2 mg naloxone D) 5 to 10 mg diazepam

D) 5 to 10 mg diazepam Feedback: A patient that has been having a seizure continuously for more than 5 min utes is said to be in status epilepticus. After ensuring airway patency and adequate ventilation. your next priority should be to pharmacologically terminate the seizure. Benzodiazepines, such as diazepam (Valium) and lorazepam (Ativan), a re effective in terminating seizures. The correct dose of diazepam for seizure termination is 5 to IO mg. If vascular access cannot be obtained, administer the benzodiazepine via the intramuscular route. If you suspect a narcotic overdose as the underlying cause of the seizure, given naloxone (Narcan) accordingly. If hypoglycemia is documented, administer 25 g of 50% dextrose. Thiamine (vitami n BI) is not routinely given to patients who are experiencing a seizure, although it should be considered if you r patient has a history of alcoholism and is hypoglycemic.

The clinical presentation of thyroid storm resembles which of the following cond itions? A) Myxedema B) Ketoacidosis C) Opioid overdose D) Amphetamine use

D) Amphetamine use Feedback: Thyroid storm is a life-threatening condition that may occur in patients with hyperthyroidism. Hyperthyroidism is a condition in which the thyroid gland produces too much of the thyroid hormones triiodothyronine (T3) and thyroxine (T4). T3 and T4 play a key role in controlling the body's basal metabolic rate (BMR). When produced in excess, these thyroid hormones produce signs and symptoms of adrenergic hyperactivity (ie, tachycardia, sweating, palpitations, anxiety). In thyroid storm, these clinical signs would be more severe, including profound tachycardia, dysrhythmias, agitation and paranoia, and cardiovascular collapse. Amphetamines al o cause adrenergic hyperactivity; of the choices listed, amphetamine u e and thyroid corm would produce the most similar clinical signs. Opioids cause central nervous system depression, resulting in a decreased level of consciousness, hypoventilation, bradycardia, and hypotension. Advanced hypothyroidism, called myxedema, results from a severe deficiency of T3 and T4. In contrast to thyroid storm, myxedema does not present with signs of adrenergic hyperactivity.

A 27-year-old female overdosed on an unknown type of drug. Her skin is hot and fl ushed and her breathing is rapid and deep. Her BP is 98/64 mm Hg and her heart rate is 120 beats/min. What should you suspect? A) Heroin B) Alcohol C) Cocaine D) Aspirin

D) Aspirin Feedback: Of the drugs listed, aspirin (acetylsalicylic acid [ASA)) overdose would be the most likely cause of the patient's clinical presentation. Aspi rin toxicity causes significant metabolic acidosis; the respiratory buffer system responds by increasing the rate and depth of breathing, resulting in a compensatory respiratory alkalosis. Hyperthennia (as evidenced by hot, nushed skin) is often associated with a pirin toxicity. Heroin overdose is unlikely. Heroin is a narcotic central nervous system depressant; overdose would result in slow, hallow breathing; hypotension, and bradycardia. Alcohol is also a central nervous system depressant; severe toxicity causes re piratory depression. An overdose of cocaine, a sympathomimetic drug, can also cause hyperthermia; however, you would expect the patient to be hypertensive and tachycardic.

A 54-year-old woman presents with an acute onset of confusion, generalized weakness, and slurred speech. Her BP is 114/60 mm Hg, pulse is 64 beats/min, respirations are 20 breaths/min, and oxygen saturation is 95%. What should you do? A) Administer oxygen B) Obtain a 12-lead ECG C) Give an IV fluid bolus D) Assess her blood glucose

D) Assess her blood glucose Feedback: Of the options listed, blood glucose assessment will quickly enable you to determine if the cause of her symptoms is hypoglycemia. If hypoglycemia is present, it should be treated without delay; the longer hypoglycemia persists, the greater the possibility of brain injury. If the patient is normoglycemic, however, then you should consider the possi bility of a stroke. Although the patient's symptoms are not con istent with a cardiac problem, it would still be prudent to obtain a 12-lead ECG because many cardiac dysrhythmia can cause neurologic symptoms due to decreased cerebral blood supply. There is no evidence of respiratory distress or hypoxemia; therefore, oxygen i s not cl in ically indicated. Her vital signs are stable, so IV fluid boluses are not indicated either.

A 30 -year-old male is in cardiac arrest after he overdosed on heroin. What should you do after initiating CPR? A) Establish vascular access and give 2 mg naloxone B) Perform intubation and ventilate at 20 breaths/min C) Establish vascular access and give 1 mg epinephrine D) Assess his cardiac rhythm and establish vascular access

D) Assess his cardiac rhythm and establish vascular access Feedback: Immediate management for cardiac arrest, regardless of the suspected cause, is to begin CPR and assess the patient's cardiac rhythm. You should then establish vascular access and administer appropriate medication therapy. Death from opioid overdose is caused by hypoxia and acidosis; therefore, the single most important therapy is high-quality CPR. Do NOT hyperventilate ANY cardiac arrest patient; doing so significantly increases imrathoracic pressure and can reduce cardiac refill during the recoil phase of compressions. Most patients who arrest from opioid overdose are in asystole or bradycardic PEA (a reflection of severe hypoxia and acidosis); VF and VT are much less common. Epinephrine is an appropriate initial medication to administer, regardles of the patient's cardiac rhythm. While evidence suggests that naloxone (Narcan) has minimal i m pact on patient outcomes, even when the cause of the arrest is an opioid overdose, any therapy that has the potential to establish ROSC is reasonable. Remember, the cause of the patient's arrest is hypoxia and acidosis, not a "naloxone deficiency."

Which of the fo llowing drugs are indicated for a pat ient with copious bronchial secretions, marked bradycardia, and profuse diaphoresis following exposure to an industrial pesticide? A) Atropine and physostigmine B) Epineph rine and 2-PAM chloride C) Pralidoxime chloride and diazepam D) Atropine and pralidoxime chloride

D) Atropine and pralidoxime chloride Feedback : Organophosphate and carbamate pesticides are cholinergic agents that stimulate the parasympathetic nervous system (PNS) by deactivating acetylcholinesterase (AChE). AChE is an enzyme that regulates the degradation of acetylcholine (ACh}, the chemical neurotransmitter of the PNS. Without AChE, /\Ch accumulates and causes profound stimulation of the PNS. The mnemonic DUMBELS is u eful for remembering the signs and symptoms of organophosphate and carbamate poisoning; it stands for defecation, urination, miosis (pupillary constriction), bradycardia and bronchorrhea, emesis, lacrimation, and sal ivation. Atropine sulfate, an anticholinergic agent, is the first-line d rug used in the treatment of organophosphate/carbamate poisoning. Atropine competitively antagonizes di e action of ACh and reverses its muscarinic effects (ie, bradycardi a, excessive secretions). Atropine is given in a dose of I to 2 mg every 5 to 15 minu tes until secretions clear and the heart rate increases; higher doses are often req uired for patients with severe toxicity. Pralidoxime chloride (2-PAM chloride, Protopam) i given after atropine to reactivate AChE, allowing i t to resume its normal function. The dose is 600 mg via autoinjector or I to 2 g IV over 1 5 to 30 minutes. Atropine and pralidoxime are commonly given together via autoinjector (Duodote). Seizures are common in patients with organophosphate/carbamate poisoning, and should be terminated with a benzodiazepine (ie, lorazepam [Ativan], diazepam [Valium], midazolam [Versed]). Physostigmine is an AChE inhibitor that interferes with the metabolism of ACh. Because it inhibits the action of AChE, physostigmine is clearly not indicated and would only exacerbate the patient' condition.

A woman is found unresponsive by her roommate. According to the roommate, the patient, who has type I diabetes, was fine 30 minutes ago. The patient has rapid, shallow respirations; pallor and diaphoresis; and a weak, thready pulse. Which of the following wou ld you expect to discover? A) Marked hyperglycemia B) Acetone or fruity odor on her breath C) Peaked T waves and wide QRS complexe D) Blood glucose level less than 60 mg/dL

D) Blood glucose level less than 60 mg/dL Feedback: This patient's cl inical presentation-pallor, diaphoresis, tachypnea, tachycardia, unresponsiveness-is classic for insulin shock (hypoglycemic crisis). The fact that she was fine 30 minutes ago indicates that her condition occurred acutely, another hallmark feature of insulin shock. When you assess her blood glucose level, you should anticipate that it will be below 60 mg/dL (hypoglycemia). Unlike in uli n hock, diabetic ketoacidosis (OKA) progresses lowly, over 12 to 48 hours, and is the result of hyperglycemia. In OKA, blood gl ucose levels of 400 mg/dL or higher are not u ncommon. Other signs of OKA include deep, rapid respirations w ith a fruity or acetone breath odor (Kussmaul respirations), which are a response of the respiratory system to eliminate ketoacids from the body; warm, dry skin, which is the result of dehydration secondary to the osmotic diuresis caused by hyperglycemia; and a rapid, full pulse, which is also the result of dehydration. Changes in serum potassium are possible in patients with OKA. The associated osmotic diuresis may cause dangerously low potassium levels (hypokalemia), resulting in marked cardiac instability. On d1e ECG, hypokalemia may manifest with flattened T waves, p rom inent U waves, ST segment depression, and prolongation of the PR and QT intervals.

A man presents with a sudden painless loss of vision in one eye. He has no other symptoms. What should you suspect? A) Papilledema B) Conjunctivitis C) Acute narrow-angle glaucoma D) Central retinal arterial occlusion

D) Central retinal arterial occlusion Feedback : Central retinal arterial occlusion is a condition in which the blood supply to the retina becomes blocked because of a thrombus or embolus in the central retinal artery or one of its branches. Possible causes of this condition include an embolus from a carotid artery, valvular heart disease, drug abuse, fat emboli, arterial spasm, or contraceptive use. Central retinal arterial occlusion presents with a sudden painless loss of vision in one eye. If the central retinal artery is blocked, then the patient will likely experience complete vision loss. If a branch of the central retinal artery i s blocked, then the patient will likely develop partial vision loss. Glaucoma is a group of conditions that lead to increased intraocular pressure. It i one of the leading causes of blindne s. One type of glaucoma is called narrow-angle glaucoma (also called angle-closure glaucoma). In this condition, access to the drainage channel is narrowed, which prevents proper drainage of aqueous h u mor. Pressure builds up i n the posterior chamber of the eye, which pushes die lens forward. The lens then pushes the iris into the drainage channel, completely blocking it. A patient with an acute narrow-angle glaucoma attack may complain of severe eye pain, headache, photophobia, nausea, and vomiting. Conjunctivitis, also called "pink eye is a condition in which the conjunctiva becomes in0amed and red. Vision loss is not common with conjunctivitis. Papilledema results from swelling or in0ammation of the optic nerve at the posterior part of die eye. The optic nerve communicates between the eyes and the brain. When it is affected, patients may experience headaches, nausea and vomiting, temporary vision loss, or narrowing visual fields. They may also experience a graying in their field of vision.

Diarrhea, marked bradycardia, miosis, and hypersalivation are consistent with which type of toxidrome? A) Opioid B) Sympatholytic C) Amphetamine D) Cholinergic

D) Cholinergic Feedback: The syndrome-like signs and symptoms of a poisonous agent are called a toxidrome. Toxidromes are useful for remembering the assessment and management of different substances that fall u nder the same clinical umbrella. Cholinergic agents (ie, Diazinon, orthene, sarin, tabun) stimulate the parasympathetic nervous system by deactivating acetylcholinesterase, an enzyme that regulates the degradation of acetylcholine. The signs and symptoms of cholinergic toxici ty-that is, the toxidrome-can be remembered with the mnemonic DUMBELS, which stands for defecation, urination, miosis (pupillary constriction), bradycardia and bronchorrhea, emesis, lacrimation, and salivation. Opioid (narcotic) toxicity presents with bradycardia, respiratory depression, hypotension, and pupillary constriction. Sympatholytic (ie, alpha or beta blocker) toxicity presents wid1 bradycardia, hypotension, and hypoglycemia, among other signs. Amphetamine (upper) toxicity presents with restlessness, pupillary dilation, tachycardia, hypertension, tachypnea, and insomnia, among other signs.

A man is under the infl uence of an unknown substance. He is agitated, his BP is 180/100 mm Hg, his pulse rate is 150 beats/min, and his skin is hot to the touch. What should you suspect? A) Heroin B) Valium C) Alcohol D) Cocaine

D) Cocaine Feedback: The patient's clinical presentation of agitation, hypertension, tachycardia, and hyperthermia are consistent with overdose of a sympathomimetic drug. Of the options listed, cocaine is the only sympathomimetic. Heroin, diazepam (Valium), and alcohol are all central nervous system depressants; signs and symptoms of central nervous system depressant overdose include lethargy or coma, hypotension, bradycardia, and hypoventilation.

Which of the fo llowing disease processes is characterized by myoclonic jerking, major cognitive deterioration, visual impairment, and ataxia? A) Pick disease B) Alzheimer disease C) Huntington disease D) Creutzfeldt-Jakob disease

D) Creutzfeldt-Jakob disease Feedback : Dementia is the chronic deterioration of memory, personality, language skills, perception, reasoning, or judgment. These changes can occur over weeks to years and can be subtle. There are several disease processes that are classified as dementia, and while there is no specific prehospital treatment for them, it is important to understand the many disease processes that affect the brain. Creutzfeldt-Jakob disease occurs when prions (proteins) clump together with resultant death of neurons. It is marked by myoclonic jerking, major cognitive deterioration, visual impairment, and unstable gait (ataxia). The typical course of the disease is 8 months, and it i always fatal. The exact cause of Pick disease is unknown but is thought to have a genetic a pect. Pick disease causes damage to neurons in the frontal and temporal lobes of the brain. It most commonly occurs in people between ages 55 and 65 years and is marked by an insidious presentation of socially inappropriate behavior, such as stealing and obsessive behaviors. The patient may be apathetic, depressed, or inappropriately elated. Additionally, rest tremors, difficulty naming common objects (anomia), and incontinence may be present. The typical course of the disease is 6 years. The cause of Alzheimer disease is multi factorial and is thought to occur because of a gradual accumulation of plaques in the brai n, which causes neuronal death, and an eventual decrease in brain mass. These processes begin 10 to 20 years before symptoms appear. Chronic, insidious memory loss is the earliest finding.

A woman with a history of hypertension presents with an acute onset of tearing abdominal pain. She is conscious and alert , but restless. Her BP is 86/56 mm Hg, pulse rate is 120 beats/min, and respirations are 28 breaths/min. What should you do? A) Administer 100 mcg fentanyl to relieve her pain B) Increase her blood p ressure with IV fl uid boluses C) Vigorously palpate her abdomen for a p ulsating mas D) Determine if her femoral pulses are of equal strength

D) Determine if her femoral pulses are of equal strength Feedback: Your patient may be experiencing acute dissection of the abdominal aorta. A unilateral femoral pulse deficit (one femoral pulse is weaker than the other) may be observed in patients with dissection of the abdominal aorta, especially if the dissection occurs where the common iliac arteries branch. Vigorous palpation is not indicated for anyone with abdominal pain; it only causes further pain and anxiety, and could aggravate the patient's condition-especially if an aortic dissection is present. The goal of IV fluid therapy is to maintain adequate perfusion; this is most reliably assessed by noting the patient's mental status and q ual ity of peripheral pulses. The blood pressure needed to maintain adequate perfusion varies from person to person. Establ ish at least one large-bore IV and be prepared to give fluid boluses if needed (ie, mental status decreases, BP falls profoundly, peripheral pulses markedly weaken). Narcotic analgesia (ie, morphine, fentanyl) may be given to patients with severe abdominal pain; however, this patient is hypotensive, so narcotic analgesia is contraindicated.

Which of the following is a defining factor in the transition from human immunodeficiency virus (HIV) infection to acquired immunodeficiency syndrome (AIDS)? A) Antibodies are detected in the blood B) Fever that lasts greater than 7 days C) Transient increase in the T-cell count D) Development of opportunistic infections

D) Development of opportunistic infections Feedback: Acquired immunodeficiency syndrome (AIDS) is the end-stage disease process caused by infection with the human immunodeficiency virus (HIV). AltJ1ough it can obviously be stated that all patients with AI DS are H IV positive, it cannot be stated that all HIV-positive patients have AIDS. A diagnosis of AIDS is made when the number of T-helper lymphocytes (CD4 cells) fall below a certain level, or when the patient develops one of a specific group of opportunistic infections (also called AIDS-defining or AIDS-related conditions). Such opportunistic infections include Pneumocystis carinii pneumonia, cycomegalovirus, red or purple malignant skin lesions called Kaposi sarcoma, atypical tuberculosis, and cryptococcol meningitis. The point at which antibodies to the virus are detected in the blood is called seroconversion; this usually occurs within the fi rst 3 months following exposure to HIV. Wi th antiretroviral therapy, patients can remain HIV-positive for many years without showing any evidence of infection.

Shortly after his dialysis treatment, a 66-year-old man presents with con fusion, a headache, and nausea. What sho uld you suspect? A) Interstitial nephritis B) Acute air embolism C) Severe hyperkalemia D) Disequilibrium syndrome

D) Disequilibrium syndrome Feedback: Dialysis rapidly lowers the concentration of urea in the blood, whereas the concentration of solutes in the cerebrospinal fluid (CSF) remains high. Water moves by osmosis from a solution of lower solute concentration into a solution of higher solute concentration. Thus, as a consequence of dialysis, water initially shifts from the bloodstream into the CSF, which mildly increases intracranial pressure. If this occurs, the patient may experience disequilibrium syndrome, a condition characterized by nausea, vomiting, headache, and confusion. After a few hours, the fluid will re-equilibrate between the blood and CSF, and the patient's symptoms will resolve on their own. Intersti tial nephritis, a cause of i ntra renal acu te renal failure, is caused by chronic inflammation of the interstitial cells sun-ounding the nephrons; this condition would not explain the patient's symptoms. Acute air embolism may occur if air enters any of the fittings or connections in the dialysis system; signs and symptoms include acute dyspnea, hypotension, and cyanosis. Hyperkalemia, an increase in serum potassium, would most likely occur if a patient missed a dialysis treatment; signs and symptoms include profound muscle weakness, QRS widening, and peaked T waves.

Which of the following regulates a person's level of consciousness? A) Diencephalon B) Limbic system C) Medulla oblongata D) Reticular activating system

D) Reticular activating system Feedback: A person's level of consciousness is regulated by the reticular activating system (RAS), which is at the level of the midbrain, a part of the brainstem. The thalamic region of the diencephalon acts as a relay center; it filters important signals from routine signals. The hypod1alamic region of the diencephalon regulates functions such as emotions (pleasure), body temperature, and interaction w ith the endocrine system. The limbic system is the region of the brain where the emotions of rage and anger are generated. The medulla oblongata is part of the brainstem. It regulates functions such as heart rate, re piratory rate, and blood pressure. The medulla and the pons (portion of the brainstem that regulates respiratory patterning and depth) function as the respiratory centers of the brain.

Which of the following patient presentations meets all components of the quick Sepsis-related Organ Failure Assessment (qSOFA) criteria for sepsis? A) GCS, 15; respiratory rate, 20 breaths/min; systolic BP, 110 mm Hg B) GCS, 13; respiratory rate, 10 breaths/min; systo lic BP, 80 mm Hg C) GCS, 15, respiratory rate, 22 breaths/min; systolic BP, 120 mm Hg D) GCS, 14; respiratory rate, 24 breaths/min; systolic BP, 90 mm Hg

D) GCS, 14; respiratory rate, 24 breaths/min; systolic BP, 90 mm Hg Feedback: Sepsis is the body's overreaction to an infection or virus that can progress to shock. People identified as being at increased risk for sepsis include those 65 years of age and older, infants younger than I year of age, people with compromised immune systems, and d1ose with chronic medical conditions (ie, heart disease, diabetes). Early identification of patients with sepsis and the prompt emergency care in the initial hours after sepsis develops improves outcomes. The quick Sepsis-related Organ Failure Assessment (qSOFJ\) is a rapid scoring system that can be used to identify patients who have an infection (or a source for infection) and may be at risk of dying from sepsis. The objective of early identification of sepsis is to begin treatment before the syndrome worsens. The qSOFJ\ score is easy to calculate because it only has three components, each of which are easily identified: (!) respiratory rate equal to or greater than 22 breaths/min, (2) Glasgow Coma Scale score less than 15, and (3) systolic BP less than or equal to 110 mm Hg. The score ranges from O to 3 points. J\ score of 2 or more is associated with poor outcomes due to sepsis. The patient with a GCS of 14, a respiratory rate of 24 breaths/min, and a systolic BP of 90 mm Hg meets all three components of the qSOFA scoring system.

A 71-year-old female presents with confusion and slurred speech. According to her husband, she was recently diagnosed with a urinary tract infection. She has type I I diabetes and hypertension and currently takes Glucophage, Zestril , and Bactrim. H e r blood glucose level reads "high;' her pulse is 1 2 0 beats/min, and h e r respirations are 20 breaths/min and regular. What should you suspect? A) Hyperglycemic ketoacidosi B) Primary adrenal insufficiency C) Severe dehydration due to infection D) Hyperosmolar hyperglycemic syndrome

D) Hyperosmolar hyperglycemic syndrome Feedback : Hyperosmolar hyperglycemic syndrome (HHS), also referred to as hyperosmolar nonketotic coma ( HONK), most often seen in patients with type II diabetes, is marked by hyperglycemia, hyperosmolarity, and an absence of significant ketosi . Most patients present with severe dehydration and focal or global neurologic deficits. H H S/HONK often develops in patients who have a secondary illness, usually an infection, that leads to reduced fl uid intake. Hyperglycemia and hyperosmolarity lead to osmotic diuresis, a fluid sh ift to the intravascular space, and intracellular dehydration. The patient in this scenario is dehydrated, but not because of her urinary tract infection. Unlike diabetic ketoacidosis (DKA), H H S/HONK does not cause ketoacidosis: this would explain the absence of deep, rapid breathing in this patient. Factors that may explain absent ketoacidosis include limited ketogenesis (ketone production due to fatty acid breakdown), low free fatty acid levels, and the availability of insulin in amounts sufficient to inhibit ketogenesis but insufficient to prevent hyperglycemia. Blood glucose levels greater than 600 mg/dL are common with H HS/HONK. The patient's presentation is not consistent with primary adrenal insufficiency (Addison disease), symptoms of which includes weight loss, vomiting, diarrhea, muscle and joint pain, abdominal pain, and increased skin pigmentation.

What does a high level of thyroid-stimulating hormone (TSH) in the blood indicate? A) Graves disease B) Cushing syndrome C) Elevated T3/T4 levels D) Hypothyroidism

D) Hypothyroidism Feedback : Thyroid-stimulating hormone (TSH), a hormone produced by the pituitary gland, stimulates the thyroid gland to produce triiodothyronine (T3) and thyroxine (T4). If the thyroid produces too little of these key hormones, TSH levels will increase in an effort LO stimulate the thyroid. Thus, an elevated TSH level indicates hypothyroidism. If the thyroid produces excess T3 and T4 , as is the case with Graves disease (hyperthyroidism), TSH levels would be correspondingly low. Cushing syndrome is not a disease of the thyroid; it is caused by an excess of cortisol production by the adrenal glands or by excessive use of corti ol or imilar corticosteroid (glucocorticoid) hormones, such as prednisone, hydrocortisone, or methylprednisolone.

By which of the following mechanisms does cyanide ca use hypoxia? A) It destroys red blood cells. B) It makes excess carbon dioxide. C) It binds to the hemoglobin molecule. D) It prevents the metabolism of oxygen.

D) It prevents the metabolism of oxygen. Feedback: Cyanide causes cellular hypoxia by blocking oxidative phosphorylation in the mitochondria and preventing oxygen metabolism. If oxygen cannot be metabolized, then carbon dioxide cannot be made. Cyanide does not destroy the red blood cells. Carbon monoxide (CO) causes hypoxia by binding tightly to the hemoglobin molecule, much more so than oxygen, which prevents oxygen from reaching the cellular level.

Following prolonged dehydration, a 67-year-old man presents with hypotension, tachycardia, and oliguria. He has no past medical history. What should you suspect? A) Chronic renal failure B) Intrarenal acute renal failure C) Postrenal acute renal failure D) Prerenal acute renal failure

D) Prerenal acute renal failure Feedback : Acute renal failure (ARF) is a sudden decrease in glomerular filtration rate (GFR), which causes toxins to accumulate in the blood. ARF is classified into three types, based on the area in which the failure occurs: prerenal, intrarenal, and postrenal. Prerenal ARF is caused by hypope,fusion of the kid neys; not enough blood passes i nto the glomeruli for them to produce filtrate. The most common causes of prerenal ARF are hypovolemia (blood loss or severe dehydration), trauma, sepsis, shock, and heart failure. lntrarenal ARF involves damage to one of three areas in the kidney: the glomeruli capillaries and small blood vessels, the cells of the renal tubules, or the renal parenchyma. This type of ARF can be caused by immune- mediated diseases (ie, type I diabetes) or by certain medications. Postrenal ARF is caused by obstruct ion of urine flow from the kidneys. The source of this obstruction is often a blockage of the urethra by an enlarged prostate gland, renal calculi (kidney stones), or strictures. As a result, pressure in the nephrons is increased, which causes them to stop functioning. Chronic renal fail ure (CRF) is progressive and irreversible inadequate kidney function caused by the permanent loss of nephrons. This disease develops over months or years. Most cases of CRF are caused by systemic disease, such as hypertension or diabetes.

A man is confused and has slurred speech after working in the heat for several ho urs. His skin is hot and moist, his pulse is rapid and weak, and his respirations are rapid and shallow. What should you do? A) Establish an IV and ad mini ter 25 grams of 50% dextrose B) Establish an IV line and administer a 20 -mL/kg fluid bol us C) Loosen his clothes and allow his body to cool down gradually D) Remove his clothes and apply ice packs to his groin and axillae

D) Remove his clothes and apply ice packs to his groin and axillae Feedback: The patient is experiencing exertional heatstroke and needs immediate active cooling. Do not let the fact that his skin is moist fool you; residual perspiration can remain on the skin, even though the patient is not actively sweating. The major factors that should clue you into heatstroke are hot skin and abnormal mentation. After moving the patient out of the hot environment, you should remove die patient's clothing and apply ice packs to his groin and axillae. The high body temperatures associated with heat troke can cause permanent brain damage or death if not promptly treated. Establish IV access and administer crystalloid fluid boluses as dictated by the patient's blood pressure. If hypoglycemia is confirmed by gl ucometer, administer 12.5 to 25 g of 50% dextrose. Transport the patient without delay and continue active cooling measures en route.

Four hours after starting a new prescription of Zoloft, a woman presents with deliri um, tachycardia, and hypertension. Her muscles are rigid, she has trismus, and she is diaphoretic. What should you suspect? A) Rhabdomyolysis B) Salicylate toxicity C) Dystonic reaction D) Serotonin syndrome

D) Serotonin syndrome Feedback : The patient's clin ical presentation is rather classic for serotonin syndrome. Serotonin syndrome is an idiosyncratic complication that occasionally occurs with antidepressant therapy. This condition is not limited to selective serotonin reuptake inhibitors (SSRis); it can also occur when patients take any combination of drugs that increase central serotonin neurotransmission. Drugs that can cause serotonin yndrome include SSRls (ie, sertraline [Zoloft], fluoxetine (Prozac], paroxetine (Paxil]), monoamine oxidase inhibitors, or MAO Is (ie, isocarboxazid (Marplan], phenelzine (Nardi!), tranylcypromine [Pamate]), tricyclic antidepressants (ie, amitriptyl ine [Elavil], nortriptyline (Pamelor]), lithium, stimulants, and opioids. The onset of serotonin syndrome is commonly within 6 hours of ingestion (and possibly the result of something as simple as an increased dosage of a medication). Signs and symptoms include delirium, tachycardia, hypertension, diaphoresis, diarrhea, muscle rigidity, myoclonus, hyperreflexia, and trismus. Prehospital treatment is supportive; if hyperthermia is present, consider cool ing measures. There is no indication of salicylate (aspirin) toxicity. Some of her symptoms could also be caused by a dystonic reaction; however, delirium is not a common sign, and d rug- induced dyston ia is most common following use of antipsychotic drugs. Rhabdomyolysis, the destruction of muscle leading to a release of potassium and myoglobin, is not typically seen with antidepressant therapy; it is more common following stimulant use, such as cocaine or methamphetamine.

A 39-year-old man is in cardiac arrest and has a core body temperature of 81° F (27.2°C). Which of the following standard resuscitation interventions may require modification? A) Cardiac monitoring B) Airway management C) Chest compressions D) Vasoactive medications

D) Vasoactive medications Feedback: Current ACLS resuscitation guidelines state that if a patient with severe hypothermia (CBT less than 82°F [28°CJ) is pulseless and apneic, CPR should be initiated without delay. just as you would for a normothermic patient, and cardiac rhythm identification should be rapidly accomplished. If the patient is in a shockable rhythm. defibrillate and immediately resume CPR. If the rhythm persists after a single shock, it may be reasonable to perform further defibrillation attempts with simultaneous rewarming strategies. Airway management, as a procedure, is no different i n the hypOlhermic patient than i t is in the normothermic patient. Current ACLS resuscitation guidelines suggest that "it may be reasonable to consider administration of a va opressor during cardiac arrest according to the standard ACLS algorithm concurrent with rewarming strategies." However, the 2014 Wilderness Medicine Society (WMS) practice guidelines recommend that providers "not administer any vasoactive drug until the patient has been rewarmed to 86°F (30°cr Because of these conflicting recommendations, th e administration of vasoactive drugs during hypothermic cardiac arrest would be the most likely to require modification.


संबंधित स्टडी सेट्स

A03 Practice Quiz for Alteryx Interactive Lessons (53 Questions)

View Set

Microsoft Excel and Access 1.07, 1.08, and 1.09

View Set

ATI Fundamentals large file study ****

View Set

Biology 106 Chapter 20: Genes Within Population

View Set

Chapter 21- The Immune System Part 1

View Set